Chuyên đề bồi dưỡng học sinh giỏi - toán 7

118 17 0
Chuyên đề bồi dưỡng học sinh giỏi - toán 7

Đang tải... (xem toàn văn)

Tài liệu hạn chế xem trước, để xem đầy đủ mời bạn chọn Tải xuống

Thông tin tài liệu

Vận dụng kiến thức vào các bài tập về xác định hệ số của luỹ thừa bậc n của một nhị thức, vận dụng vào các bài toán phân tích đa thức thành nhân tử.. B. KIẾN THỨC VÀ BÀI TẬP VẬN DỤNG:.[r]

(1)

20 CHUYÊN ĐỀ BỒI DƢỠNG TOÁN

CHUYÊN ĐỀ - PHẤN TÍCH ĐA THỨC THÀNH NHÂN TỬ A MỤC TIÊU:

* Hệ thống lại dạng toán phƣơng pháp phân tích đa thức thành nhân tử * Giải số tập phân tích đa thức thành nhân tử

* Nâng cao trình độ kỹ phân tích đa thức thành nhân tử

B CÁC PHƢƠNG PHÁP VÀ BÀI TẬP

I TÁCH MỘT HẠNG TỬ THÀNH NHIỀU HẠNG TỬ:

Định lí bổ sung:

+ Đa thức f(x) có nghiệm hữu tỉ có dạng p/q p ƣớc hệ số tự do, q ƣớc dƣơng hệ số cao

+ Nếu f(x) có tổng hệ số f(x) có nhân tử x –

+ Nếu f(x) có tổng hệ số hạng tử bậc chẵn tổng hệ số hạng tử bậc lẻ f(x) có nhân tử x +

+ Nếu a nghiệm nguyên f(x) f(1); f(- 1) khác f(1)

a - f(-1)

a + số nguyên

Để nhanh chóng loại trừ nghiệm ƣớc hệ số tự

1 Ví dụ 1: 3x2 – 8x +

Cách 1: Tách hạng tử thứ

3x2 – 8x + = 3x2 – 6x – 2x + = 3x(x – 2) – 2(x – 2) = (x – 2)(3x – 2)

Cách 2: Tách hạng tử thứ nhất:

3x2 – 8x + = (4x2 – 8x + 4) - x2 = (2x – 2)2 – x2 = (2x – + x)(2x – – x) = (x – 2)(3x – 2)

Ví dụ 2: x3 – x2 -

Ta nhân thấy nghiệm f(x) có x =   1; 2; 4, có f(2) = nên x = nghiệm

của f(x) nên f(x) có nhân tử x – Do ta tách f(x) thành nhóm có xuất nhân tử x –

Cách 1:

x3 – x2 – =  2     2 

2 2 ( 2) 2( 2)

xxxxx x x x x  x =   

2

(2)

Cách 2: 3    

4 8 ( 2)( 4) ( 2)( 2)

xx  x  x   x   x   xxx  x x

=    

2 ( 2) ( 2)( 2)

x  xx  x  xx  x

Ví dụ 3: f(x) = 3x3 – 7x2 + 17x –

Nhận xét:  1, không nghiệm f(x), nhƣ f(x) khơng có nghiệm ngun Nên f(x) có nghiệm nghiệm hữu tỉ

Ta nhận thấy x =

3 nghiệm f(x) f(x) có nhân tử 3x – Nên

f(x) = 3x3 – 7x2 + 17x – = 2  2    

3xx 6x 2x15x 5 3xx  6x 2x  15x5

= 2

(3 1) (3 1) 5(3 1) (3 1)( 5) x x  x x  x  xxx

Vì 2

2 ( 1) ( 1)

xx  xx   x   với x nên khơng phân tích đƣợc thành nhân tử

Ví dụ 4: x3 + 5x2 + 8x +

Nhận xét: Tổng hệ số hạng tử bậc chẵn tổng hệ số hạng tử bậc lẻ nên đa thức có nhân tử x +

x3 + 5x2 + 8x + = (x3 + x2 ) + (4x2 + 4x) + (4x + 4) = x2(x + 1) + 4x(x + 1) + 4(x + 1) = (x + 1)(x2 + 4x + 4) = (x + 1)(x + 2)2

Ví dụ 5: f(x) = x5 – 2x4 + 3x3 – 4x2 +

Tổng hệ số nên đa thức có nhân tử x – 1, chia f(x) cho (x – 1) ta có: x5 – 2x4 + 3x3 – 4x2 + = (x – 1)(x4 - x3 + 2x2 - 2x - 2)

Vì x4 - x3 + 2x2 - 2x - khơng có nghiệm ngun khơng có nghiệm hữu tỉ nên khơng phân tích đƣợc

Ví dụ 6: x4 + 1997x2 + 1996x + 1997 = (x4 + x2 + 1) + (1996x2 + 1996x + 1996) = (x2 + x + 1)(x2 - x + 1) + 1996(x2 + x + 1)

= (x2 + x + 1)(x2 - x + + 1996) = (x2 + x + 1)(x2 - x + 1997)

Ví dụ 7: x2 - x - 2001.2002 = x2 - x - 2001.(2001 + 1)

= x2 - x – 20012 - 2001 = (x2 – 20012) – (x + 2001) = (x + 2001)(x – 2002)

(3)

20 CHUYÊN ĐỀ BỒI DƢỠNG TOÁN

1 Thêm, bớt số hạng tử để xuất hiệu hai bình phƣơng: Ví dụ 1: 4x4 + 81 = 4x4 + 36x2 + 81 - 36x2 = (2x2 + 9)2 – 36x2

= (2x2 + 9)2 – (6x)2 = (2x2 + + 6x)(2x2 + – 6x) = (2x2 + 6x + )(2x2 – 6x + 9)

Ví dụ 2: x8 + 98x4 + = (x8 + 2x4 + ) + 96x4

= (x4 + 1)2 + 16x2(x4 + 1) + 64x4 - 16x2(x4 + 1) + 32x4

= (x4 + + 8x2)2 – 16x2(x4 + – 2x2) = (x4 + 8x2 + 1)2 - 16x2(x2 – 1)2 = (x4 + 8x2 + 1)2 - (4x3 – 4x )2

= (x4 + 4x3 + 8x2 – 4x + 1)(x4 - 4x3 + 8x2 + 4x + 1)

2 Thêm, bớt số hạng tử để xuất nhân tử chung

Ví dụ 1: x7 + x2 + = (x7 – x) + (x2 + x + ) = x(x6 – 1) + (x2 + x + )

= x(x3 - 1)(x3 + 1) + (x2 + x + ) = x(x – 1)(x2 + x + ) (x3 + 1) + (x2 + x + 1) = (x2 + x + 1)[x(x – 1)(x3 + 1) + 1] = (x2 + x + 1)(x5 – x4 + x2 - x + 1)

Ví dụ 2: x7 + x5 + = (x7 – x ) + (x5 – x2 ) + (x2 + x + 1) = x(x3 – 1)(x3 + 1) + x2(x3 – 1) + (x2 + x + 1)

= (x2 + x + 1)(x – 1)(x4 + x) + x2 (x – 1)(x2 + x + 1) + (x2 + x + 1)

= (x2 + x + 1)[(x5 – x4 + x2 – x) + (x3 – x2 ) + 1] = (x2 + x + 1)(x5 – x4 + x3 – x + 1)

Ghi nhớ:

Các đa thức có dạng x3m +

+ x3n + + nhƣ: x7 + x2 + ; x7 + x5 + ; x8 + x4 + ; x5 + x + ; x8 + x + ; … có nhân tử chung x2 + x +

III ĐẶT BIẾN PHỤ:

Ví dụ 1: x(x + 4)(x + 6)(x + 10) + 128 = [x(x + 10)][(x + 4)(x + 6)] + 128 = (x2 + 10x) + (x2 + 10x + 24) + 128

Đặt x2

+ 10x + 12 = y, đa thức có dạng

(y – 12)(y + 12) + 128 = y2 – 144 + 128 = y2 – 16 = (y + 4)(y – 4) = ( x2 + 10x + )(x2 + 10x + 16 ) = (x + 2)(x + 8)( x2 + 10x + )

(4)

x4 + 6x3 + 7x2 – 6x + = x2 ( x2 + 6x + – + 2 x x ) = x

2

[(x2 + 2

x ) + 6(x -

x ) + ]

Đặt x -

x = y x

+ 2

x = y

2 + 2,

A = x2(y2 + + 6y + 7) = x2(y + 3)2 = (xy + 3x)2 = [x(x -

x )

+ 3x]2 = (x2 + 3x – 1)2

Chú ý: Ví dụ giải cách áp dụng đẳng thức nhƣ sau: A = x4 + 6x3 + 7x2 – 6x + = x4 + (6x3 – 2x2 ) + (9x2 – 6x + )

= x4 + 2x2(3x – 1) + (3x – 1)2 = (x2 + 3x – 1)2

Ví dụ 3: A = 2 2

(xyz )(x y z) (xyyz+zx)

= 2 2 2

(x y z ) 2(xy yz+zx) (x y z ) (xy yz+zx)

         

 

Đặt 2

xyz = a, xy + yz + zx = b ta có

A = a(a + 2b) + b2 = a2 + 2ab + b2 = (a + b)2 = ( 2

xyz + xy + yz + zx)2

Ví dụ 4: B = 4 2 2 2 2

2(xyz ) ( xyz ) 2(xyz )(x y z)   (x y z)

Đặt x4

+ y4 + z4 = a, x2 + y2 + z2 = b, x + y + z = c ta có:

B = 2a – b2 – 2bc2 + c4 = 2a – 2b2 + b2 - 2bc2 + c4 = 2(a – b2) + (b –c2)2 Ta lại có: a – b2

= - 2( 2 2 2

x yy zz x ) b –c2 = - 2(xy + yz + zx) Do đó; B = - 4( 2 2 2

x yy zz x ) + (xy + yz + zx)2

= 2 2 2 2 2 2 2

4x y 4y z 4z x 4x y 4y z 4z x 8x yz 8xy z 8xyz 8xyz x( y z)

           

Ví dụ 5: 3 3

(a b c  ) 4(a  b c ) 12 abc

Đặt a + b = m, a – b = n 4ab = m2

– n2 a3 + b3 = (a + b)[(a – b)2 + ab] = m(n2 +

2

m - n

4 ) Ta có:

C = (m + c)3 –

3

3 2

m + 3mn

4c 3c(m - n )

4   = 3( - c

3

+mc2 – mn2 + cn2)

= 3[c2(m - c) - n2(m - c)] = 3(m - c)(c - n)(c + n) = 3(a + b - c)(c + a - b)(c - a + b)

III PHƢƠNG PHÁP HỆ SỐ BẤT ĐỊNH:

Ví dụ 1: x4

(5)

20 CHUYÊN ĐỀ BỒI DƢỠNG TOÁN

Nhận xét: số 1, 3 không nghiệm đa thức, đa thức khơng có nghiệm ngun củng khơng có nghiệm hữu tỉ

Nhƣ đa thức phân tích đƣợc thành nhân tử phải có dạng

(x2 + ax + b)(x2 + cx + d) = x4 + (a + c)x3 + (ac + b + d)x2 + (ad + bc)x + bd

đồng đa thức với đa thức cho ta có:

6 12 14

a c ac b d ad bc bd

   

    

   

 

Xét bd = với b, d  Z, b   1, 3 với b = d = hệ điều kiện trở thành

6

8

3 14

3 a c

ac c c

a c ac a

bd

   

        

  

        

 

 

Vậy: x4

- 6x3 + 12x2 - 14x + = (x2 - 2x + 3)(x2 - 4x + 1)

Ví dụ 2: 2x4 - 3x3 - 7x2 + 6x +

Nhận xét: đa thức có nghiệm x = nên có thừa số x - ta có: 2x4 - 3x3 - 7x2 + 6x + = (x - 2)(2x3 + ax2 + bx + c)

= 2x4 + (a - 4)x3 + (b - 2a)x2 + (c - 2b)x - 2c 

4

1

2

5

4

a

a

b a

b c b

c c

   

     

   

   

   

  

Suy ra: 2x4 - 3x3 - 7x2 + 6x + = (x - 2)(2x3 + x2 - 5x - 4) Ta lại có 2x3

+ x2 - 5x - đa thức có tổng hệ số hạng tử bậc lẻ bậc chẵn nahu nên có nhân tử x + nên 2x3

+ x2 - 5x - = (x + 1)(2x2 - x - 4) Vậy: 2x4

- 3x3 - 7x2 + 6x + = (x - 2)(x + 1)(2x2 - x - 4) Ví dụ 3:

(6)

12

4 10

3

3

6 12

2

3 12

ac

a bc ad

c c a

b bd

d d b

 

 

    

 

   

   

   

  

  

 12x2 + 5x - 12y2 + 12y - 10xy - = (4 x - 6y + 3)(3x + 2y - 1)

BÀI TẬP:

Phân tích đa thức sau thành nhân tử:

CHUYấN ĐỀ - SƠ LƢỢC VỀ CHỈNH HỢP,

1) x3 - 7x +

2) x3 - 9x2 + 6x + 16 3) x3 - 6x2 - x + 30 4) 2x3 - x2 + 5x + 5) 27x3 - 27x2 + 18x - 6) x2 + 2xy + y2 - x - y - 12

7) (x + 2)(x +3)(x + 4)(x + 5) - 24 8) 4x4 - 32x2 +

9) 3(x4 + x2 + 1) - (x2 + x + 1)2

10) 64x4 + y4

11) a6 + a4 + a2b2 + b4 - b6 12) x3 + 3xy + y3 -

13) 4x4 + 4x3 + 5x2 + 2x + 14) x8 + x +

15) x8 + 3x4 +

(7)

20 CHUYÊN ĐỀ BỒI DƢỠNG TOÁN

CHUYÊN ĐỀ 2: HOÁN VỊ, TỔ HỢP A MỤC TIÊU:

* Bƣớc đầu HS hiểu chỉnh hợp, hoán vị tổ hợp

* Vận dụng kiến thức vào ssó toán cụ thể thực tế * Tạo hứng thú nâng cao kỹ giải toán cho HS

B KIẾN THỨC: I Chỉnh hợp:

1 định nghĩa: Cho tập hợp X gồm n phần tử Mỗi cách xếp k phần tử tập hợp X (  k  n) theo thứ tự định gọi chỉnh hợp chập k n phần tử Số tất chỉnh hợp chập k n phần tử đƣợc kí hiệu k

n

A

2 Tính số chỉnh chập k n phần tử

II Hoán vị:

1 Định nghĩa: Cho tập hợp X gồm n phần tử Mỗi cách xếp n phần tử tập hợp X theo thứ tự định gọi hoán vị n phần tử

Số tất hốn vị n phần tử đƣợc kí hiệu Pn

2 Tính số hốn vị n phần tử ( n! : n giai thừa)

III Tổ hợp:

1 Định nghĩa: Cho tập hợp X gồm n phần tử Mỗi tập X gồm k phần tử n phần tử tập hợp X (  k  n) gọi tổ hợp chập k n phần tử Số tất tổ hợp chập k n phần tử đƣợc kí hiệu k

n

C

2 Tính số tổ hợp chập k n phần tử

k n

A = n(n - 1)(n - 2)…[n - (k - 1)]

k n

C = Ann : k! = n(n - 1)(n - 2) [n - (k - 1)]

k!

Pn =

n n

(8)

C Ví dụ:

1 Ví dụ 1:

Cho chữ số: 1, 2, 3, 4,

a) có số tự nhiên có ba chữ số, chữ số khác nhau, lập ba chữ số

b) Có số tự nhiên có chữ số, chữ số khác nhau, lập chữ số c)Có cách chọn ba chữ số chữ số

Giải:

a) số tự nhiên có ba chữ số, chữ số khác nhau, lập ba chữ số chỉnh hợp chập phần tử:

5

A = 5.(5 - 1).(5 - 2) = = 60 số

b) số tự nhiên có chữ số, chữ số khác nhau, lập chữ số hoán vị cua phần tử (chỉnh hợp chập phần tử):

5

A = 5.(5 - 1).(5 - 2).(5 - 3).(5 - 4) = = 120 số

c) cách chọn ba chữ số chữ số tổ hợp chập phần tử: C35 = 5.(5 - 1).(5 - 2) 60 10

3! 3.(3 - 1)(3 - 2)   nhóm

2 Ví dụ 2:

Cho chữ số 1, 2, 3, 4, Dùng chữ số này:

a) Lập đƣợc số tự nhiên có chữ số khơng có chữ số lặp lại? Tính tổng số lập đƣợc

b) lập đƣợc số chẵn có chữ số khác nhau?

c) Lập đƣợc số tự nhiên có chữ số, hai chữ số kề phải khác

d) Lập đƣợc số tự nhiên có chữ số, chữ số khác nhau, có hai chữ số lẻ, hai chữ số chẵn

(9)

20 CHUYÊN ĐỀ BỒI DƢỠNG TOÁN

a) số tự nhiên có chữ số, chữ số khác nhau, lập chữ số chỉnh hợp chập phần tử:

5

A = 5.(5 - 1).(5 - 2).(5 - 3) = = 120 số Trong hang (Nghìn, trăm, chục, đơn vị), chữ số có mặt: 120 : = 24 lần Tổng chữ số hang: (1 + + + + 5) 24 = 15 24 = 360

Tổng số đƣợc lập: 360 + 3600 + 36000 + 360000 = 399960 b) chữ số tận có cách chọn (là 4)

bốn chữ số trƣớc hoán vị của chữ số cịn lại có P4 = 4! = = 24 cách

chọn

Tất có 24 = 48 cách chọn

c) Các số phải lập có dạng abcde, : a có cách chọn, b có cách chọn (khác a),

c có cách chọn (khác b), d có cách chọn (khác c), e có cách chọn (khác d) Tất có: = 1280 số

d) Chọn chữ số chẵn, có cách chọn

chọn chữ số lẻ, có cách chọn Các chữ số hốn vị, có: 4! =1 = 72 số

Bài 3: Cho 

xAy 180 Trên Ax lấy điểm khác A, Ay lấy điểm khác A 12

điểm nói (kể điểm A), hai điểm củng đƣợc nối với đoạn thẳng Có tam giác mà đỉnh 12 điểm

Giải

Cách 1: Tam giác phải đếm gồm ba loại:

+ Loại 1: tam giác có đỉnh A, đỉnh thứ thuộc Ax (có cách chọn), đỉnh thứ thuộc Ay (có cách chọn), gồm có: = 30 tam giác

+ Loại 2: Các tam giác có đỉnh điểm B1,

B2, B3, B4, B5 (có cách chọn), hai đỉnh

điểm A1, A2, A3, A4, A5, A6 ( Có

2

6.5 30 15 2!

C    cách chọn) Gồm 15 = 75 tam giác

x y B5

B4 B2

B1

A5

A4

A3

A6 B3

(10)

+ Loại 3: Các tam giác có đỉnh điểm A1, A2, A3, A4, A5, A6 hai đỉnh

trong điểm B1, B2, B3, B4, B5 gồm có:

2

5.4 20

6 60

2!

C    tam giác Tất có: 30 + 75 + 60 = 165 tam giác

Cách 2: số tam giác chọn 12 điểm 12

12.11.10 1320 1320 220

3! 3.2

C    

Số ba điểm thẳng hang điểm thuộc tia Ax là:

7.6.5 210 210 35 3! 3.2 C    

Số ba điểm thẳng hang điểm thuộc tia Ay là:

6.5.4 120 120 20 3! 3.2 C    

Số tam giác tạo thành: 220 - ( 35 + 20) = 165 tam giác

D BÀI TẬP:

Bài 1: cho số: 0, 1, 2, 3, từ chữ số lập đƣợc số tự nhiên:

a) Có chữ số gồm chữ số ấy? b) Có chữ số, có chữ số khác nhau? c) có chữ số, chữ số khác nhau?

d) có chữ số, chữ số giống nhau?

Bài 2: Có số tự nhiên có chữ số lập chữ số 1, 2, biết số chia

hết cho

(11)

20 CHUYÊN ĐỀ BỒI DƢỠNG TOÁN

CHUYÊN ĐỀ - LUỸ THỪA BẬC N CỦA MỘT NHỊ THỨC A MỤC TIÊU:

HS nắm đƣợc công thức khai triển luỹ thừa bậc n nhị thức: (a + b)n

Vận dụng kiến thức vào tập xác định hệ số luỹ thừa bậc n nhị thức, vận dụng vào tốn phân tích đa thức thành nhân tử

B KIẾN THỨC VÀ BÀI TẬP VẬN DỤNG:

I Nhị thức Niutơn:

Trong đó: k n

n(n - 1)(n - 2) [n - (k - 1)] C

1.2.3 k

II Cách xác định hệ số khai triển Niutơn: Cách 1: Dùng công thức k

n

n(n - 1)(n - 2) [n - (k - 1)] C

k !

Chẳng hạn hệ số hạng tử a4

b3 khai triển (a + b)7

4

7.6.5.4 7.6.5.4

C 35

4! 4.3.2.1

  

Chú ý: a) k n

n ! C

n!(n - k) !

 với quy ƣớc 0! = 

7! 7.6.5.4.3.2.1

C 35

4!.3! 4.3.2.1.3.2.1

  

b) Ta có: k n

C = C k - n nên

4

7

7.6.5

C C 35

3!

  

2 Cách 2: Dùng tam giác Patxcan

Đỉnh 1

Dòng 1(n = 1) 1 1

Dòng 2(n = 1) 1 2 1

Dòng 3(n = 3) 1 3 3 1

Dòng 4(n = 4) 1 4 6 4 1

Dòng 5(n = 5) 1 5 10 10 5 1

Dòng 6(n = 6) 1 6 15 20 15 6 1

Trong tam giác này, hai cạnh bên gồm số 1; dòng k + đƣợc thành lập từ dòng k (a + b)n = an +

n

C an - b + n

C an - b2 + …+ n n

(12)

(k 1), chẳng hạn dịng (n = 2) ta có = + 1, dòng (n = 3): = + 1, = + dòng (n = 4): = + 3, = + 3, = + 1, …

Với n = thì: (a + b)4

= a4 + 4a3b + 6a2b2 + 4ab3 + b4 Với n = thì: (a + b)5

= a5 + 5a4b + 10a3b2 + 10a2b3 + 5ab4 + b5 Với n = thì: (a + b)6

= a6 + 6a5b + 15a4b2 + 20a3b3 + 15a2 b4 + 6ab5 + b6

3 Cách 3:

Tìm hệ số hạng tử đứng sau theo hệ số hạng tử đứng trƣớc: a) Hệ số hạng tử thứ

b) Muốn có hệ số của hạng tử thứ k + 1, ta lấy hệ số hạng tử thứ k nhân với số mũ biến hạng tử thứ k chia cho k

Chẳng hạn: (a + b)4 = a

4

+ 1.4

1 a

b + 4.3

2 a

b2 + 4.3.2

2.3 ab

+ 4.3.2

2.3.4 b

Chú ý rằng: hệ số khai triển Niutơn có tính đối xứng qua hạng tử đứng giữa, nghĩa hạng tử cách hai hạng tử đầu cuối có hệ số

(a + b)n = an + nan -1b + n(n - 1)

1.2 a n -

b2 + …+ n(n - 1)

1.2 a

bn - + nan - 1bn - + bn

III Ví dụ:

1 Ví dụ 1: phân tích đa thức sau thành nhân tử a) A = (x + y)5 - x5 - y5

Cách 1: khai triển (x + y)5

rút gọn A

A = (x + y)5 - x5 - y5 = ( x5 + 5x4y + 10x3y2 + 10x2y3 + 5xy4 + y5) - x5 - y5 = 5x4y + 10x3y2 + 10x2y3 + 5xy4 = 5xy(x3 + 2x2y + 2xy2 + y3)

= 5xy [(x + y)(x2 - xy + y2) + 2xy(x + y)] = 5xy(x + y)(x2 + xy + y2) Cách 2: A = (x + y)5 - (x5 + y5)

x5 + y5 chia hết cho x + y nên chia x5 + y5 cho x + y ta có:

x5 + y5 = (x + y)(x4 - x3y + x2y2 - xy3 + y4) nên A có nhân tử chung (x + y), đặt (x + y) làm nhân tử chung, ta tìm đƣợc nhân tử lại

(13)

20 CHUYÊN ĐỀ BỒI DƢỠNG TOÁN = 7x6y + 21x5y2 + 35x4y3 + 35x3y4 + 21x2y5 + 7xy6

= 7xy[(x5 + y5 ) + 3(x4y + xy4) + 5(x3y2 + x2y3 )]

= 7xy {[(x + y)(x4 - x3y + x2y2 - xy3 + y4) ] + 3xy(x + y)(x2 - xy + y2) + 5x2y2(x + y)} = 7xy(x + y)[x4 - x3y + x2y2 - xy3 + y4 + 3xy(x2 + xy + y2) + 5x2y2 ]

= 7xy(x + y)[x4 - x3y + x2y2 - xy3 + y4 + 3x3y - 3x2y2 + 3xy3 + 5x2y2 ]

= 7xy(x + y)[(x4 + 2x2y2 + y4) + 2xy (x2 + y2) + x2y2 ] = 7xy(x + y)(x2 + xy + y2 )2

Ví dụ 2:Tìm tổng hệ số đa thức có đƣợc sau khai triển

a) (4x - 3)4

Cách 1: Theo cônh thức Niu tơn ta có:

(4x - 3)4 = 4.(4x)3.3 + 6.(4x)2.32 - 4x 33 + 34 = 256x4 - 768x3 + 864x2 - 432x + 81 Tổng hệ số: 256 - 768 + 864 - 432 + 81 =

b) Cách 2: Xét đẳng thức (4x - 3)4 = c0x

+ c1x

+ c2x

+ c3x + c4

Tổng hệ số: c0 + c1 + c2 + c3 + c4

Thay x = vào đẳng thức ta có: (4.1 - 3)4

= c0 + c1 + c2 + c3 + c4

Vậy: c0 + c1 + c2 + c3 + c4 =

* Ghi chú: Tổng hệ số khai triển nhị thức, đa thức giá trị đa thức x =

C BÀI TẬP:

Bài 1: Phân tích thành nhân tử

a) (a + b)3 - a3 - b3 b) (x + y)4 + x4 + y4

(14)

CHUÊN ĐỀ - CÁC BÀI TOÁN VỀ SỰ CHIA HẾT CỦA SỐ NGUYÊN A MỤC TIÊU:

* Củng cố, khắc sâu kiến thức toán chia hết số, đa thức

* HS tiếp tục thực hành thành thạo toán chứng minh chia hết, không chia hết, sốnguyên tố, số phƣơng…

* Vận dụng thành thạo kỹ chứng minh chia hết, không chia hết… vào toán cụ thể

B.KIẾN THỨC VÀ CÁC BÀI TOÁN: I Dạng 1: Chứng minh quan hệ chia hết 1 Kiến thức:

* Để chứng minh A(n) chia hết cho số m ta phân tích A(n) thành nhân tử có nhân tử làm bội m, m hợp số ta lại phân tích thành nhân tử có đoi nguyên tố nhau, chứng minh A(n) chia hết cho số

* Chú ý:

+ Với k số nguyên liên tiếp củng tồn bội k

+ Khi chứng minh A(n) chia hết cho m ta xét trƣờng hợp số dƣ chia A(n) cho m

+ Với số nguyên a, b số tự nhiên n thì:

2 Bài tập:

2 Các toán

Bài 1: chứng minh

a) 251 - chia hết cho b) 270 + 370 chia hết cho 13

c) 1719 + 1917 chi hết cho 18 d) 3663 - chia hết cho nhƣng không chia hết cho 37

+) an - bn chia hết cho a - b (a - b) +) a2n + + b2n + chia hết cho a + b + (a + b)n = B(a) + bn

(15)

20 CHUYÊN ĐỀ BỒI DƢỠNG TOÁN e) 24n -1 chia hết cho 15 với n N

Giải

a) 251 - = (23)17 -  23 - =

b) 270 + 370 (22)35 + (32)35 = 435 + 935  + = 13 c) 1719 + 1917 = (1719 + 1) + (1917 - 1)

1719 +  17 + = 18 1917 -  19 - = 18 nên (1719 + 1) + (1917 - 1) hay 1719 + 1917  18

d) 3663 -  36 - = 35 

3663 - = (3663 + 1) - chi cho 37 dƣ - e) 4n - = (24) n -  24 - = 15

Bài 2: chứng minh

a) n5 - n chia hết cho 30 với n  N ;

b) n4 -10n2 + chia hết cho 384 với n lẻ n Z c) 10n +18n -28 chia hết cho 27 với n N ;

Giải:

a) n5 - n = n(n4 - 1) = n(n - 1)(n + 1)(n2 + 1) = (n - 1).n.(n + 1)(n2 + 1) chia hết cho (n - 1).n.(n+1) tích ba số tự nhiên liên tiếp nên chia hết cho (*)

Mặt khác n5

- n = n(n2 - 1)(n2 + 1) = n(n2 - 1).(n2 - + 5) = n(n2 - 1).(n2 - ) + 5n(n2 - 1) = (n - 2)(n - 1)n(n + 1)(n + 2) + 5n(n2 - 1)

Vì (n - 2)(n - 1)n(n + 1)(n + 2) tích số tự nhiên liên tiếp nên chia hết cho 5n(n2 - 1) chia hết cho

Suy (n - 2)(n - 1)n(n + 1)(n + 2) + 5n(n2 - 1) chia hết cho (**) Từ (*) (**) suy đpcm

b) Đặt A = n4

-10n2 + = (n4-n2 ) - (9n2 - 9) = (n2 - 1)(n2 - 9) = (n - 3)(n - 1)(n + 1)(n + 3) Vì n lẻ nên đặt n = 2k + (k  Z)

(16)

Từ (1) (2) suy A chia hết cho 16 24 = 384 c) 10 n +18n -28 = ( 10

n

- 9n - 1) + (27n - 27) + Ta có: 27n - 27  27 (1)

+ 10 n - 9n - = [(

n

9 + 1) - 9n - 1] = 

n

9 - 9n = 9( 

n

1 - n)  27 (2)  

n

1 - n  

n

1 - n số có tổng chữ số chia hết cho Từ (1) (2) suy đpcm

3 Bài 3: Chứng minh với số nguyên a a) a3 - a chia hết cho

b) a7 - a chia hết cho Giải

a) a3 - a = a(a2 - 1) = (a - 1) a (a + 1) tích ba số nguyên liên tiếp nên tồn số bội nên (a - 1) a (a + 1) chia hết cho

b) ) a7 - a = a(a6 - 1) = a(a2 - 1)(a2 + a + 1)(a2 - a + 1) Nếu a = 7k (k  Z) a chia hết cho

Nếu a = 7k + (k Z) a2 - = 49k2 + 14k chia hết cho

Nếu a = 7k + (k Z) a2 + a + = 49k2 + 35k + chia hết cho Nếu a = 7k + (k Z) a2 - a + = 49k2 + 35k + chia hết cho Trong trƣờng hợp củng có thừa số chia hết cho

Vậy: a7

- a chia hết cho

Bài 4: Chứng minh A = 13 + 23 + 33 + + 1003 chia hết cho B = + + + + 100 Giải

Ta có: B = (1 + 100) + (2 + 99) + + (50 + 51) = 101 50

Để chứng minh A chia hết cho B ta chứng minh A chia hết cho 50 101 Ta có: A = (13 + 1003) + (23 + 993) + +(503 + 513)

(17)

20 CHUYÊN ĐỀ BỒI DƢỠNG TỐN Lại có: A = (13

+ 993) + (23 + 983) + + (503 + 1003)

Mỗi số hạng ngoặc chia hết cho 50 nên A chia hết cho 50 (2) Từ (1) (2) suy A chia hết cho 101 50 nên A chi hết cho B

Bài tập nhà

Chứng minh rằng: a) a5 – a chia hết cho

b) n3 + 6n2 + 8n chia hết cho 48 với n chẵn c) Cho a l số nguyên tố lớn Cmr a2

– chia hết cho 24 d) Nếu a + b + c chia hết cho a3

+ b3 + c3 chia hết cho e) 20092010 không chia hết cho 2010

f) n2 + 7n + 22 không chia hết cho

Dạng 2: Tìm số dƣ phép chia Bài 1:

Tìm số dƣ chia 2100

a)cho 9, b) cho 25, c) cho 125 Giải

a) Luỹ thừa sát với bội 23

= = -

Ta có : 2100 = (23)33 = 2.(9 - 1)33 = 2.[B(9) - 1] = B(9) - = B(9) + Vậy: 2100

chia cho dƣ b) Tƣơng tự ta có: 2100

= (210)10 = 102410 = [B(25) - 1]10 = B(25) + Vậy: 2100

chia chop 25 dƣ c)Sử dụng cơng thức Niutơn:

2100 = (5 - 1)50 = (550 - 549 + … + 50.49

2

- 50 ) +

Không kể phần hệ số khai triển Niutơn 48 số hạng đầu chứa thừa số với số mũ lớn nên chia hết cho 53

= 125, hai số hạng tiếp theo: 50.49

2

- 50.5 chia hết cho 125 , số hạng cuối

Vậy: 2100

(18)

Bài 2:

Viết số 19951995

thành tổng số tự nhiên Tổng lập phƣơng chia cho dƣ bao nhiêu?

Giải

Đặt 19951995

= a = a1 + a2 + …+ an

Gọi 3 3

1 n

S  a a + a + + a = 3 3

1 n

a a + a + + a + a - a = (a1

3

- a1) + (a2

- a2) + …+ (an

- an) + a

Mỗi dấu ngoặc chia hết cho dấu ngoặc tích ba số tự nhiên liên tiếp Chỉ cần tìm số dƣ chia a cho

1995 số lẻ chia hết cho 3, nên a củng số lẻ chia hết cho 3, chia cho dƣ

Bài 3: Tìm ba chữ số tận 2100 viết hệ thập phân giải

Tìm chữ số tận tìm số dƣ phép chia 2100

cho 1000 Trƣớc hết ta tìm số dƣ phép chia 2100

cho 125 Vận dụng ta có 2100

= B(125) + mà 2100 số chẵn nên chữ số tận 126, 376, 626 876

Hiển nhiên 2100

chia hết cho 2100 = 1625 chi hết ba chữ số tận chia hết cho

trong số 126, 376, 626 876 có 376 chia hết cho Vậy: 2100

viết hệ thập phân có ba chữ số tận 376

Tổng quát: Nếu n số chẵn không chia hết cho chữ số tận 376

Bài 4: Tìm số dƣ phép chia số sau cho

a) 2222 + 5555 b)31993 c) 19921993 + 19941995 d) 21930

3

Giải

a) ta có: 2222 + 5555 = (21 + 1)22 + (56 – 1)55 = (BS +1)22 + (BS – 1)55 = BS + + BS - = BS nên 2222 + 5555 chia dƣ

b) Luỹ thừa sát với bội 33

(19)

20 CHUYÊN ĐỀ BỒI DƢỠNG TOÁN Ta thấy 1993 = BS + = 6k + 1, đó:

31993= 6k + = 3.(33)2k = 3(BS – 1)2k = 3(BS + 1) = BS + c) Ta thấy 1995 chia hết cho 7, đó:

19921993 + 19941995 = (BS – 3)1993 + (BS – 1)1995 = BS – 31993 + BS – Theo câu b ta có 31993 = BS + nên

19921993 + 19941995 = BS – (BS + 3) – = BS – nên chia cho dƣ d) 21930

3 = 32860 = 33k + = 3.33k = 3(BS – 1) = BS – nên chia cho dƣ

Bài tập nhà

Tìm số d ƣ khi: a) 21994 cho

b) 31998 + 51998 cho 13

c) A = 13 + 23 + 33 + + 993 chia cho B = + + + + 99

Dạng 3: Tìm điều kiện để xảy quan hệ chia hết

Bài 1: Tìm n  Z để giá trị biểu thức A = n3 + 2n2 - 3n + chia hết cho giá trị biểu thức B = n2

- n Giải

Chia A cho B ta có: n3 + 2n2 - 3n + = (n + 3)(n2 - n) + Để A chia hết cho B phải chia hết cho n2

- n = n(n - 1) chia hết cho n, ta có:

n - -

n - - -

n(n - 1) 2

loại loại

Vậy: Để giá trị biểu thức A = n3

+ 2n2 - 3n + chia hết cho giá trị biểu thức B = n2 - n n   1; 2

Bài 2:

(20)

Giải

Ta có: n5 +  n3 +  n2(n3 + 1) - (n2 - 1)  n3 +  (n + 1)(n - 1)  n3 +  (n + 1)(n - 1)  (n + 1)(n2 - n + 1)  n -  n2 - n + (Vì n +  0) a) Nếu n = 1

Nếu n > n - < n(n - 1) + < n2

- n + nên xẩy n -  n2 - n + Vậy giá trụ n tìm đƣợc n =

b) n -  n2 - n +  n(n - 1)  n2 - n +  (n2 - n + ) -  n2 - n +   n2 - n + Có hai trƣờng hợp xẩy ra:

+ n2 - n + =  n(n - 1) =  n

n

   

 (Tm đề bài)

+ n2 - n + = -1  n2 - n + = (Vơ nghiệm)

Bài 3: Tìm số ngun n cho:

a) n2 + 2n -  11 b) 2n3 + n2 + 7n +  2n - c) n4 - 2n3 + 2n2 - 2n +  n4 - d) n3 - n2 + 2n +  n2 + Giải

a) Tách n2 + 2n - thành tổng hai hạng tử có hạng tử B(11) n2 + 2n -  11  (n2 - 2n - 15) + 11  11 (n - 3)(n + 5) + 11  11

 (n - 3)(n + 5)  11 n 1 n = B(11) +

n + 1 n = B(11) -

 

 

 

 

b) 2n3 + n2 + 7n + = (n2 + n + 4) (2n - 1) +

Để 2n3

+ n2 + 7n +  2n -  2n - hay 2n - Ƣ(5)

2n = - n = - 2n = -1 n = 2n = n = 2n = n =

 

  

 

  

  

 

Vậy: n   2; 0; 1;  2n3 + n2 + 7n +  2n - c) n4 - 2n3 + 2n2 - 2n +  n4 -

Đặt A = n4

- 2n3 + 2n2 - 2n + = (n4 - n3) - (n3 - n2) + (n2 - n) - (n - 1)

(21)

20 CHUYÊN ĐỀ BỒI DƢỠNG TOÁN

A chia hết cho b nên n    A chia hết cho B  n -  n +  (n + 1) -  n +

  n + 

 n = -3 n = -

n = - n = -

n = n =

n = n = (khong Tm)

 

  

  

 

  

 

Vậy: n   3;  2;  n4 - 2n3 + 2n2 - 2n +  n4 -

d) Chia n3 - n2 + 2n + cho n2 + đƣợc thƣơng n - 1, dƣ n + Để n3

- n2 + 2n +  n2 + n +  n2 +  (n + 8)(n - 8)  n2 + 65  n2 + Lần lƣợt cho n2

+ 1; 5; 13; 65 ta đƣợc n 0; 2; 8 Thử lại ta có n = 0; n = 2; n = (T/m)

Vậy: n3

- n2 + 2n +  n2 + n = 0, n =

Bài tập nhà:

Tìm số nguyên n để: a) n3 – chia hết cho n –

b) n3 – 3n2 – 3n – chia hết cho n2 + n + c)5n – 2n chia hết cho 63

Dạng 4: Tồn hay không tồn chia hết Bài 1: Tìm n  N cho 2n – chia hết cho Giải

Nếu n = 3k ( k  N) 2n – = 23k – = 8k - chia hết cho

Nếu n = 3k + ( k  N) 2n – = 23k + – = 2(23k – 1) + = BS + Nếu n = 3k + ( k  N) 2n – = 23k + – = 4(23k – 1) + = BS + V ậy: 2n

– chia hết cho n = BS

Bài 2: Tìm n  N để: a) 3n – chia hết cho

b) A = 32n + + 24n + chia hết cho 25 c) 5n – 2n chia hết cho

(22)

a) Khi n = 2k (k N) 3n – = 32k – = 9k – chia hết cho – = Khi n = 2k + (k N) 3n – = 32k + – = (9k – ) + = BS + Vậy : 3n

– chia hết cho n = 2k (k N)

b) A = 32n + + 24n + = 27 32n + 2.24n = (25 + 2) 32n + 2.24n = 25 32n + 2.32n + 2.24n = BS 25 + 2(9n + 16n)

Nếu n = 2k +1(k N) 9n + 16n = 92k + + 162k + chia hết cho + 16 = 25

Nếu n = 2k (k N) 9n có chữ số tận , cịn 16n có chữ số tận suy 2((9n + 16n) có chữ số tận nên A không chia hết không chia hết cho 25

c) Nếu n = 3k (k N) 5n – 2n = 53k – 23k chia hết cho 53 – 23 = 117 nên chia hết cho Nếu n = 3k + 5n – 2n = 5.53k – 2.23k = 5(53k – 23k) + 23k = BS + 8k

= BS + 3(BS – 1)k = BS + BS + Tƣơng tự: n = 3k + 5n

(23)

20 CHUYÊN ĐỀ BỒI DƢỠNG TỐN

CHUN ĐỀ 5: SỐ CHÍNH PHƢƠNG

I Số phƣơng: A Một số kiến thức:

Số phƣơng: số bình phƣơng số khác Ví dụ:

4 = 22; = 32

A = 4n2 + 4n + = (2n + 1)2 = B2

+ Số phƣơng khơng tận chữ số: 2, 3, 7,

+ Số phƣơng chia hết cho chia hết cho 4, chia hết cho chia hết cho 9, chia hết cho chia hết cho 25, chia hết cho 23

chia hết cho 24,… + Số 

n

11 = a 

n

99 = 9a 9a + = 

n

99 + = 10n

B Một số toán:

1 Bài 1:

Chứng minh rằng: Một số phƣơng chia cho 3, cho dƣ Giải

Gọi A = n2 (n N)

a) xét n = 3k (k N)  A = 9k2 nên chia hết cho n = 3k  (k N)  A = 9k2  6k + 1, chia cho dƣ Vậy: số phƣơng chia cho dƣ

b) n = 2k (k N) A = 4k2 chia hết cho

n = 2k +1 (k N) A = 4k2 + 4k + chia cho dƣ Vậy: số phƣơng chia cho dƣ

Chú ý: + Số phƣơng chẵn chia hết cho

+ Số phƣơng lẻ chia cho dƣ 1( Chia củng dƣ 1) Bài 2: Số số sau số phƣơng

(24)

b) N = 19922 + 19932 + 19942 + 19952 c) P = + 9100 + 94100 + 1994100

d) Q = 12 + 22 + + 1002 e) R = 13 + 23 + + 1003 Giải

a) số 19932, 19942 chia cho dƣ 1, 19922 chia hết cho  M chia cho dƣ M khơng số phƣơng

b) N = 19922 + 19932 + 19942 + 19952 gồm tổng hai số phƣơng chẵn chia hết cho 4, hai số phƣơng lẻ nên chia dƣ suy N khơng số phƣơng

c) P = + 9100 + 94100 + 1994100 chia dƣ nên khơng số phƣơng d) Q = 12 + 22 + + 1002

Số Q gồm 50 số phƣơng chẵn chia hết cho 4, 50 số phƣơng lẻ, số chia dƣ nên tổng 50 số lẻ chia dƣ Q chia dƣ nên Q khơng số phƣơng

e) R = 13 + 23 + + 1003 Gọi Ak = + + + k =

k(k + 1)

2 , Ak – = + + + k =

k(k - 1)

Ta có: Ak2 – Ak -12 = k3 đó:

13 = A1

23 = A2

– A1

n3 = An2 = An - 12

Cộng vế theo vế đẳng thức ta có: 13 + 23 + +n3 = An

2

=  

2

2

n(n + 1) 100(100 1)

50.101

2

    

   

    số phƣơng

3 Bài 3:

(25)

20 CHUYÊN ĐỀ BỒI DƢỠNG TOÁN A = (

n

11 1)(10 n+1 + 5) +

1

1

10

.(10 5) 10 n n       

Đặt a = 10n+1 A = a -

9 (a + 5) + =

2

2

a + 4a - + a + 4a + a +

9

 

   

 

b) B =

n

111 1

n -

555 56 ( cĩ n số n-1 số 5)

B =

n

111 1

n

555 5 + =

n

111 1 10n +

n

555 5 + =

n

111 1 10n +

n

111

 

 

 + Đặt

n

11 1 = a 10n = 9a + nên

B = a(9a + 1) + 5a + = 9a2 + 6a + = (3a + 1)2=  n -

33

c) C =

2n

11 1.+ 44

n

+ Đặt a =

n

11 1 Thì C =

n

11 1

n

11 1 +

n

11 1 + = a 10n + a + a +

= a(9a + 1) + 5a + = 9a2 + 6a + = (3a + 1)2 d) D =

n

99 98

n

00 01 Đặt

n

99 9 = a  10n = a + D =

n

99 9 10n + + 10n + + = a 100 10n + 80 10n +

= 100a(a + 1) + 80(a + 1) + = 100a2 + 180a + 81 = (10a + 9)2 = (

n +

99 )2 e) E =

n

11 1

n +

22 5 =

n

11 1

n +

22 200 + 25 =

n

11 1.10n + +

n

11 100 + 25

= [a(9a + 1) + 2a]100 + 25 = 900a2 + 300a + 25 = (30a + 5)2 = (

n

33 35)2 f) F =

100

44  =

100

11 1 số phƣơng

100

11 1 số phƣơng Số

100

11 1 số lẻ nên số phƣơng chia cho phải dƣ Thật vậy: (2n + 1)2 = 4n2 + 4n + chia dƣ

100

(26)

vậy

100

11 1 khơng số phƣơng nên F =

100

44

 khơng số phƣơng Bài 4:

a) Cho số A =

2m

11 11 ; B =

m +

11 11 ; C =

m

66 66  CMR: A + B + C + số phƣơng

Ta có: A

2

10

m

; B =

1

10

9

m 

; C = 6.10

m

Nên:

A + B + C + =

2

10

m +

1

10

9

m 

+ 6.10

m

+ =

2

10 10 6(10 1) 72

m  m   m 

=

2

10 10.10 6.10 72

m  m  m 

=  

2 2

10 16.10 64 10 8

9

mmm

  

  

 

b) CMR: Với x,y Ỵ Z A = (x+y)(x+2y)(x+3y)(x+4y) + y4

số phƣơng A = (x2 + 5xy + 4y2) (x2 + 5xy + 6y2) + y4

= (x2 + 5xy + 4y2) [(x2 + 5xy + 4y2) + 2y2) + y4

= (x2 + 5xy + 4y2)2 + 2(x2 + 5xy + 4y2).y2 + y4 = [(x2 + 5xy + 4y2) + y2)2 = (x2 + 5xy + 5y2)2

Bài 5: Tìm số nguyên dƣơng n để biểu thức sau số phƣơng a) n2 – n + b) n5 – n +

Giải

a) Với n = n2 – n + = khơng số phƣơng Với n = n2 – n + = số phƣơng

Với n > n2 – n + khơng số phƣơng Vì (n – 1)2 = n2 – (2n – 1) < n2 – (n - 2) < n2

b) Ta có n5 – n chia hết cho Vì n5 – n = (n2 – 1).n.(n2 + 1)

Với n = 5k n chia hết cho

(27)

20 CHUYÊN ĐỀ BỒI DƢỠNG TOÁN

Nên n5 – n + chia cho dƣ nên n5 – n + có chữ số tận nên n5 – n + khơng số phƣơng

Vậy : Khơng có giá trị n thỗ mãn tốn

Bài :

a)Chứng minh : Mọi số lẻ viết đƣợc dƣới dạng hiệu hai số phƣơng b) Một số phƣơng có chữ số tận chữ số hàng chục chữ số chẵn Giải

Mọi số lẻ có dạng a = 4k + a = 4k +

Với a = 4k + a = 4k2 + 4k + – 4k2 = (2k + 1)2 – (2k)2

Với a = 4k + a = (4k2 + 8k + 4) – (4k2 + 4k + 1) = (2k + 2)2 – (2k + 1)2 b)A số phƣơng có chữ số tận nên

A = (10k  3)2 =100k2  60k + = 10.(10k2 6) + Số chục A 10k2  số chẵn (đpcm)

Bài 7:

Một số phƣơng có chữ số hàng chục chữ số lẻ Tìm chữ số hàng đơn vị Giải

Gọi n2 = (10a + b)2 = 10.(10a2 + 2ab) + b2 nên chữ số hàng đơn vị cần tìm chữ số tận b2

Theo đề , chữ số hàng chục n2 chữ số lẻ nên chữ số hàng chục b2 phải lẻ Xét giá trị b từ đến có b2 = 16, b2 = 36 có chữ số hàng chục chữ số lẻ, chúng tận

Vậy : n2 có chữ số hàng đơn vị

Bài tập nhà:

Bài 1: Các số sau đây, số số phƣơng a) A =

50

22 24 b) B = 11115556 c) C =

n

99 

n

00 025 d) D =

n

44  

n -

88 89 e) M =

2n

11 –

n

22

(28)

a) n3 – n + b) n4 – n +

Bài 3: Chứng minh

a)Tổng hai số phƣơng lẻ khơng số phƣơng

(29)

20 CHUYÊN ĐỀ BỒI DƢỠNG TOÁN

29

CHUYÊN ĐỀ - CÁC BÀI TỐN VỀ ĐỊNH LÍ TA-LÉT A.Kiến thức:

1 Định lí Ta-lét:

* Định lí Ta-lét: ABC

MN // BC

 

  

AM AN

=

AB AC

* HƢ qu¶: MN // BC  AM = AN MN AB AC  BC B Bài tập áp dụng:

1 Bài 1:

Cho tứ giác ABCD, đƣờng thẳng qua A song song với BC cắt BD E, đƣờng thẳng qua B song song với AD cắt AC G

a) chứng minh: EG // CD

b) Giả sử AB // CD, chứng minh AB2 = CD EG Giải

Gọi O giao điểm AC BD a) Vì AE // BC  OE = OA

OB OC (1)

BG // AC  OB = OG OD OA (2)

Nhân (1) với (2) vế theo vế ta có: OE = OG

OD OC  EG // CD

b) Khi AB // CD EG // AB // CD, BG // AD nên

2

AB OA OD CD AB CD

= = AB CD EG

EG OG  OB AB EG AB  Bài 2:

Cho ABC vuông A, Vẽ phía ngồi tam giác tam giác ABD vuông cân B, ACF vuông cân C Gọi H giao điểm

AB CD, K giao điểm Ac BF Chứng minh rằng:

N M

C B

A

H

F K

D

A O

G E

D C

B

(30)

a) AH = AK b) AH2 = BH CK Giải

Đặt AB = c, AC = b

BD // AC (cùng vng góc với AB)

nên AH AC b AH b AH b

HB BD  c HB  c HB + AH b + c

Hay AH b AH b AH b.c

AB  b + c c  b + c b + c (1)

AB // CF (cùng vng góc với AC) nên AK AB c AK c AK c

KC  CF  b KC  b KC + AK  b + c

Hay AK b AK c AK b.c

AC  b + c b b + c b + c (2)

Từ (1) (2) suy ra: AH = AK b) Từ AH AC b

HB BD c

AK AB c

KC  CF b suy

AH KC AH KC

HB AK  HB AH(Vì AH = AK)

 AH2 = BH KC

3 Bài 3: Cho hình bình hành ABCD, đƣờng thẳng a qua A lần lƣợt cắt BD, BC, DC theo thứ tự E, K, G Chứng minh rằng:

a) AE2 = EK EG

b) 1

AE AKAG

c) Khi đƣờng thẳng a thay đổi vị trí nhƣng qua A tích BK DG có giá trị khơng đổi

Giải

a) Vì ABCD hình bình hành K  BC nên AD // BK, theo hệ định lí Ta-lét ta có:

2

EK EB AE EK AE

= = AE EK.EG

AE ED EG AE EG  

b) Ta có: AE = DE AK DB ;

AE BE

=

AG BD nên

G b

a

E K

D C

(31)

20 CHUYÊN ĐỀ BỒI DƢỠNG TOÁN

AE AE BE DE BD 1

= AE

AK AG BD DB BD AK AG

 

       

  

1 1

AE AKAG (đpcm)

c) Ta có: BK = AB BK = a

KC CG KC CG (1);

KC CG KC CG

= =

AD DG  b DG (2)

Nhân (1) với (2) vế theo vế ta có: BK = a BK DG = ab

b DG khơng đổi (Vì a = AB; b = AD

là độ dài hai cạnh hình bình hành ABCD không đổi)

4 Bài 4:

Cho tứ giác ABCD, điểm E, F, G, H theo thứ tự chia cạnh AB, BC, CD, DA theo tỉ số 1:2 Chứng minh rằng:

a) EG = FH

b) EG vng góc với FH Giải

Gọi M, N theo thứ tự trung điểm CF, DG Ta có CM =

2 CF =

3BC 

BM

=

BC 

BE BM

= =

BA BC

EM // AC  EM BM = EM = 2AC AC  BE  (1)

Tƣơng tự, ta có: NF // BD  NF CF = NF = 2BD BDCB  (2)

mà AC = BD (3)

Từ (1), (2), (3) suy : EM = NF (a)

Tƣơng tự nhƣ ta có: MG // BD, NH // AC MG = NH =

3AC (b)

Mặt khác EM // AC; MG // BD Và AC  BD EM  MG  

EMG = 90 (4) Tƣơng tự, ta có: 

FNH = 90 (5)

Từ (4) (5) suy  

EMG = FNH = 90 (c)

Từ (a), (b), (c) suy EMG = FNH (c.g.c)  EG = FH

b) Gọi giao điểm EG FH O; EM FH P; EM FN Q

PQF = 90  QPF + QFP = 90  mà QPF = OPE   (đối đỉnh), OEP = QFP   (EMG = FNH) Q P O

N M

H F

G E

D

C B

(32)

Suy  

EOP = PQF = 90  EO  OP  EG  FH 5 Bài 5:

Cho hình thang ABCD có đáy nhỏ CD Từ D vẽ đƣờng thẳng song song với BC, cắt AC M AB K, Từ C vẽ đƣờng thẳng song song với AD, cắt AB F, qua F ta lại vẽ đƣờng thẳng song song với AC, cắt BC P Chứng minh

a) MP // AB

b) Ba đƣờng thẳng MP, CF, DB đồng quy Giải

a) EP // AC  CP = AF PB FB (1)

AK // CD  CM = DC AM AK (2)

tứ giác AFCD, DCBK la hình bình hành nên AF = DC, FB = AK (3)

Kết hợp (1), (2) (3) ta có CP CM

PB AM  MP // AB

(Định lí Ta-lét đảo) (4)

b) Gọi I giao điểm BD CF, ta có: CP CM

PB AM =

DC DC AK  FB

Mà DC DI

FB  IB (Do FB // DC) 

CP DI

PB IB IP // DC // AB (5)

Từ (4) (5) suy : qua P có hai đƣờng thẳng IP, PM song song với AB // DC nên theo tiên đề Ơclít ba điểm P, I, M thẳng hang hay MP qua giao điểm CF DB hay ba đƣờng thẳng MP, CF, DB đồng quy

6 Bài 6:

Cho ABC có BC < BA Qua C kẻ đƣờng thẳng vng gốc với tia phân giác BE

ABC; đƣờng thẳng cắt BE F cắt trung tuyến BD G Chứng minh đoạn thẳng EG bị đoạn thẳng DF chia làm hai phần

I P

F K M

D C

(33)

20 CHUYÊN ĐỀ BỒI DƢỠNG TOÁN Giải

Gọi K giao điểm CF AB; M giao điểm DF BC

KBC có BF vừa phân giác vừa đƣờng cao nên

KBC cân B  BK = BC FC = FK

Mặt khác D trung điểm AC nên DF đƣờng trung bình AKC  DF // AK hay DM // AB

Suy M trung điểm BC DF =

2AK (DF đƣờng trung bình AKC), ta có BG BK

=

GD DF( DF // BK) 

BG BK 2BK =

GD DF  AK (1)

Mổt khác CE DC - DE DC AD

DE  DE DE  DE (Vì AD = DC) 

CE AE - DE DC AD

1

DE DE  DE DE 

Hay CE AE - DE AE AB DE  DE  DE  DF (vì

AE DE=

AB

DF: Do DF // AB)

Suy CE AK + BK 2(AK + BK)

DE  DE   AK  (Do DF =

2AK) 

CE 2(AK + BK) 2BK

DE  AK   AK (2)

Từ (1) (2) suy BG

GD = CE

DE  EG // BC

Gọi giao điểm EG DF O ta có OG = OE = FO

MC MB FM

 

 

   OG = OE

Bài tập nhà

Bài 1:

Cho tứ giác ABCD, AC BD cắt O Đƣờng thẳng qua O song song với BC cắt AB E; đƣờng thẳng song song với CD qua O cắt AD F

a) Chứng minh FE // BD

b) Từ O kẻ đƣờng thẳng song song với AB, AD cắt BD, CD G H Chứng minh: CG DH = BG CH

Bài 2:

M G

K

F

D E C

B

(34)

Cho hình bình hành ABCD, điểm M thuộc cạnh BC, điểm N thuộc tia đối tia BC cho BN = CM; đƣờng thẳng DN, DM cắt AB theo thứ tự E, F

Chứng minh: a) AE2 = EB FE b) EB =

2

AN DF

 

 

  EF

CHUYÊN ĐỀ – CÁC BÀI TỐN SỬ DỤNG ĐỊNH LÍ TALÉT VÀ TÍNH CHẤT ĐƢỜNG PHÂN GIÁC

A Kiến thức:

2 Tính chất đƣờng phân giác:

ABC ,AD phân giác góc A  BD = AB

CD AC

AD’là phân giác góc ngồi A: BD' = AB

CD' AC

B Bài tập vận dụng 1 Bài 1:

Cho ABC có BC = a, AB = b, AC = c, phân giác AD a) Tính độ dài BD, CD

b) Tia phân giác BI góc B cắt AD I; tính tỉ số: AI

ID

Giải

a) AD phân giác BAC nên BD AB c

CD  ACb

D' B C

A

D C

B A

a c b

I

D C

(35)

20 CHUYÊN ĐỀ BỒI DƢỠNG TOÁN

35

 BD c BD c BD = ac

CD + BD b + c a b + c b + c

Do CD = a - ac

b + c = ab b + c

b) BI phân giác ABC nên AI AB c : ac b + c IDBD  b + c a 2 Bài 2:

Cho ABC, có B< 600 phân giác AD a) Chứng minh AD < AB

b) Gọi AM phân giác ADC Chứng minh BC > DM

Giải

a)Ta có ADB = C +   A >

  A + C

2 =

0

0

180 - B 60 

ADB > B  AD < AB

b) Gọi BC = a, AC = b, AB = c, AD = d Trong ADC, AM phân giác ta có

DM AD

=

CM AC 

DM AD DM AD

= =

CM + DM AD + AC CD AD + AC

 DM = CD.AD CD d

AD + AC b + d ; CD = ab

b + c( Vận dụng 1)  DM =

abd (b + c)(b + d)

Để c/m BC > DM ta c/m a > 4abd

(b + c)(b + d) hay (b + d)(b + c) > 4bd (1)

Thật : c > d  (b + d)(b + c) > (b + d)2  4bd Bất đẳng thức (1) đƣợc c/m

Bài 3:

Cho ABC, trung tuyến AM, tia phân giác góc AMB , AMC cắt AB, AC theo thứ tự D E

a) Chứng minh DE // BC

b) Cho BC = a, AM = m Tính độ dài DE

E

D I

A

M D B

C

(36)

c) Tìm tập hợp giao diểm I AM DE ABC có BC cố định, AM = m không đổi

d) ABC có điều kiện DE đƣờng trung bình Giải

a) MD phân giác AMB nên DA MB DB MA (1)

ME phân giác AMC nên EA MC

EC MA (2)

Từ (1), (2) giả thiết MB = MC ta suy DA EA

DB EC  DE // BC

b) DE // BC  DE AD AI

BC ABAM Đặt DE = x 

x m -

x 2 2a.m

x =

a  m  a + 2m

c) Ta có: MI =

2 DE = a.m

a + 2m không đổi  I cách M đoạn không đổi nên tập

hợp điểm I đƣờng tròn tâm M, bán kính MI = a.m

a + 2m (Trừ giao điểm với BC

d) DE đƣờng trung bình ABC  DA = DB  MA = MB  ABC vuông A

4 Bài 4:

Cho ABC ( AB < AC) phân giác BD, CE

a) Đƣờng thẳng qua D song song với BC cắt AB K, chứng minh E nằm B K

b) Chứng minh: CD > DE > BE Giải

a) BD phân giác nên

AD AB AC AE AD AE

= < =

DC BC BC EB DC  EB (1)

Mặt khác KD // BC nên AD AK

DC  KB (2)

Từ (1) (2) suy AK AE AK + KB AE + EB

KB  EB  KB  EB

E

D

M

K

C B

(37)

20 CHUYÊN ĐỀ BỒI DƢỠNG TOÁN  AB AB KB > EB

KB EB E nằm K B

b) Gọi M giao điểm DE CB Ta có CBD = KDB (Góc so le trong) KBD = KDB 

mà E nằm K B nên KDB > EDBKBD > EDB  EBD > EDB  EB < DE Ta lại có CBD + ECB = EDB + DEC     DEC>ECB DEC>DCE (Vì DCE = ECB) Suy CD > ED  CD > ED > BE

5 Bài 5:

Cho ABC với ba đƣờng phân giác AD, BE, CF Chứng minh a 1

FB FA EA EC DC DB b AB CA BC CF BE AD 1 1 1      Giải

a)AD đƣờng phân giác BAC nên ta có: DB = AB DC AC (1)

Tƣơng tự: với phân giác BE, CF ta có: EC = BC

EA BA (2) ; FA CA

=

FB CB (3)

Tửứ (1); (2); (3) suy ra: DB EC FA = AB BC CA DC EA FB AC BA CB=

b) Đặt AB = c , AC = b , BC = a , AD = da

Qua C kẻ đƣờng thẳng song song với AD , cắt tia BA H Theo ĐL Talét ta có: AD BA

CH BH 

BA.CH c.CH c

AD CH

BH BA + AH b + c

  

Do CH < AC + AH = 2b nên: a bc d

b c

 

1 1 1 1

2 2

a a

b c

d bc b c d b c

    

         

   

Chứng minh tƣơng tự ta có : 1 1

b

d a c

 

   

  Và

1 1

2

c

d a b

 

   

  Nên:

1 1 1 1 1

2

a b c

d d d b c a c a b

                       

 

1 1 1 1

2

a b c

d d d a b c

(38)

1 1 1

a b c

d d d a b c

      ( đpcm )

Bài tập nhà

Cho ABC có BC = a, AC = b, AB = c (b > c), phân giác BD, CE a) Tính độ dài CD, BE suy CD > BE

(39)

20 CHUYÊN ĐỀ BỒI DƢỠNG TOÁN

CHUYÊN ĐỀ – CHỮ SỐ TẬN CÙNG

A Kiến thức:

1 Một số tính chất: a) Tính chất 1:

+ Các số có chữ số tận 0; 1; 5; 6khi nâng lên luỹ thừa bậc chữ số tận khơng thay đổi

+ Các số có chữ số tận 4; nâng lên luỹ thừa bậc lẻ chữ số tận khơng thay đổi

+ Các số có chữ số tận 3; 7; nâng lên luỹ thừa bậc 4n (n N) chữ số tận

+ Các số có chữ số tận 2; 4; nâng lên luỹ thừa bậc 4n (n N) chữ số tận

b) Tính chất 2: Một số tự nhiên nâng lên luỹ thừa bậc 4n + (n N) chữ số tận khơng thay đổi

c) Tính chất 3:

+ Các số có chữ số tận nâng lên luỹ thừa bậc 4n + (n N) chữ số tận 7; Các số có chữ số tận nâng lên luỹ thừa bậc 4n + (n N) chữ số tận

+ Các số có chữ số tận nâng lên luỹ thừa bậc 4n + (n N) chữ số tận 8; Các số có chữ số tận nâng lên luỹ thừa bậc 4n + (n N) chữ số tận

+ Các số có chữ số tận 0; 1; 4; 5; 6; nâng lên luỹ thừa bậc 4n + (n N) chữ số tận khơng đổi

2 Một số phƣơng pháp:

+ Tìm chữ số tận x = am ta xét chữ số tận a:

(40)

* Vì am = a4n + r = a4n ar

Nếu r 0; 1; 2; chữ số tận x chữ số tận ar Nếu r 2; 4; chữ số tận x chữ số tận 6.ar

B Một số ví dụ: Bài 1:

Tìm chữ số tận a) 2436 ; 1672010

b)  79 9;  14 14

14 ;  

7

4

 

 

 

Giải

a) 2436 = 2434 + = 2434 2432

2432có chữ số tận nên chữ số tận 2436 Ta có 2010 = 4.502 + nên 1672010 = 1674 502 + = 1674.502.1672

1674.502 có chữ số tận 6; 1672 có chữ số tận nên chữ số tận 1672010 chữ số tận tích 6.9

b) Ta có:

+) 99 - = (9 – 1)(98 + 97 + + + 1) = 4k (k N)  99 = 4k + 1 79 = 74k + = 74k.7 nên có chữ số tận

1414 = (12 + 2)14 = 1214 + 12.1413.2 + + 12.12.213 + 214 chia hết cho 4, hạng tử trƣớc 214 có nhân tử 12 nên chia hết cho 4; hạng tử 214 = 47 chia hết cho hay 1414 = 4k  14 14

14 = 144k có chữ số tận

+) 56 có chữ số tận nên  56 7= 5.(2k + 1)  5.(2k + 1) – = q (k, q N)  5.(2k + 1) = 4q +   

7

4

      =

4q +

= 44q có chữ số tận chữ số tận tích

(41)

20 CHUYÊN ĐỀ BỒI DƢỠNG TOÁN

a) Luỹ thừa số hạng A chia dƣ 1(Các số hạng A có dạng n4(n – 2) + (n  {2; 3; ; 2004} ) nên số hạng A luỹ thừa có chữ số tận giống (Tính chất 2) nên chữ số tận A chữ số tận tổng số hạng Từ đến 2004 có 2003 số hạng có 2000 : 10 = 200 số hạng có chữ số tận 0,Tổng chữ số tận A

(2 + + + 9) + 199.(1 + + + 9) + + + + = 9009 có chữ số tận Vây A có chữ số tận

Bài 3: Tìm

a) Hai chữ số tận 3999;  7

7

b) Ba chữ số tận 3100 c) Bốn chữ số tận 51994 Giải

a) 3999 = 3.3998 =3 9499= 3.(10 – 1)499 = 3.(10499 – 499.10498 + +499.10 – 1) = 3.[BS(100) + 4989] = 67

77 = (8 – 1)7 = BS(8) – = 4k +   7

7 = 74k + = 73 74k = 343.( 01)4k = 43 b) 3100 = 950 = (10 – 1)50 = 1050 – 50 1049 + + 50.49

2 10

– 50.10 + = 1050 – 50 1049 + + 49

2 5000 – 500 + = BS(1000) + = 001

Chú ý:

+ Nếu n số lẻ không chi hết cho ba chữ số tận n100 001 + Nếu số tự nhiên n không chia hết cho n100 chia cho 125 dƣ HD C/m: n = 5k + 1; n = 5k +

+ Nếu n số lẻ khơng chia hết cho n101 n có ba chữ số tận nhƣ c) Cách 1: 54 = 625

Ta thấy số ( 0625)n = 0625

(42)

Ta thấy 54k – chia hết cho 54 – = (52 – 1)(52 + 1) chia hết cho 16 Ta có: 51994 = 56 (51988 – 1) + 56

Do 56 chia hết cho 54, 51988 – chia hết cho 16 nên 56(51988 – 1) chia hết cho 10000 Ta có 56= 15625

Vậy bốn chữ số tận 51994 5625 Chú ý: Nếu viết 51994 = 52 (51992 – 1) + 52

Ta có: 51992 – chia hết cho 16; nhƣng 52 không chia hết cho 54

Nhƣ toán ta cần viết 51994 dƣới dạng 5n(51994 – n – 1) + 5n ; n  1994 – n chia hết cho

C Vận dụng vào toán khác

Bài 1:

Chứng minh rằng: Tổng sau khơng số phƣơng a) A = 19k + 5k + 1995k + 1996k ( k N, k chẵn)

b) B = 20042004k + 2001 Giải

a) Ta có:

19k có chữ số tận 5k có chữ số tận 1995k có chữ số tận 1996k có chữ số tận

Nên A có chữ số tận chữ số tận tổng chữ số tận tổng + + + = 17, có chữ số tận nên khơng thể số phƣơng

b) Ta có :k chẵn nên k = 2n (n  N)

20042004k = (20044)501k = (20044)1002n = ( 6)1002n luỹ thừa bậc chẵn số có chữ số tận nên có chữ số tận nên B = 20042004k + 2001 có chữ số tận 7, B khơng số phƣơng

Bài 2:

(43)

20 CHUYÊN ĐỀ BỒI DƢỠNG TOÁN a) A = 21 + 35 + 49 + + 20038005

b) B = 23 + 37 +411 + + 20058007 Giải

a) Chữ số tận A chữ số tận tổng (2 + + + 9) + 199.(1 + + + 9) + + + = 9005 Chữ số tận A nên chia A cho dƣ

b)Tƣơng tự, chữ số tận B chữ số tận tổng

(8 + + + + + + + 9) + 199.(1 + + 9) + + + + = 9024 B có chữ số tận nên B chia dƣ

Bài tập nhà

Bài 1: Tìm chữ số tận của: 3102 ;  3

7 ; 320 + 230 + 715 - 816 Bài 2: Tìm hai, ba chữ số tận của: 3555 ;  7

2

Bài 3: Tìm số dƣ chia số sau cho 2; cho 5: a) 38; 1415 + 1514

(44)

CHUYÊN ĐỀ – ĐỒNG DƢ

A Định nghĩa:

Nếu hai số nguyên a b có số dƣ phép chia cho số tự nhiên m  ta nói a đồng dƣ với b theo mơđun m, có đồng dƣ thức: a  b (mod m)

Ví dụ:7  10 (mod 3) , 12  22 (mod 10) + Chú ý: a  b (mod m)  a – b  m B Tính chất đồng dƣ thức:

1 Tính chất phản xạ: a  a (mod m)

2 Tính chất đỗi xứng: a  b (mod m)  b  a (mod m)

3 Tính chất bắc cầu: a  b (mod m), b  c (mod m) a  c (mod m) Cộng , trừ vế: a b (mod m) a c b d (mod m)

c d (mod m)

 

   

  

Hệ quả:

a) a  b (mod m)  a + c  b + c (mod m) b) a + b  c (mod m)  a  c - b (mod m) c) a  b (mod m)  a + km  b (mod m)

5 Nhân vế : a b (mod m) ac bd (mod m) c d (mod m)

  

  

Hệ quả:

a) a  b (mod m)  ac  bc (mod m) (c  Z) b) a  b (mod m)  an  bn (mod m)

6 Có thể nhân (chia) hai vế môđun đồng dƣ thức với số nguyên dƣơng a  b (mod m)  ac  bc (mod mc)

Chẳng hạn: 11  (mod 4)  22  (mod 8) ac bc (mod m) a b (mod m)

(c, m) =

 

  

Chẳng hạn : 16 (mod 7) (mod 7) (2, 7) =

 

  

(45)

20 CHUN ĐỀ BỒI DƢỠNG TỐN

C Các ví dụ:

1 Ví dụ 1:

Tìm số dƣ chia 9294 cho 15 Giải

Ta thấy 92  (mod 15)  9294  294 (mod 15) (1)

Lại có 24  (mod 15)  (24)23 22  (mod 15) hay 294  (mod 15) (2) Từ (1) (2) suy 9294  (mod 15) tức 9294 chia 15 dƣ

2 Ví dụ 2:

Chứng minh: số có dạng 2n – 4(n  N), có vơ số số chia hết cho Thật vậy:

Từ 24  (mod 5) 24k  (mod 5) (1) Lại có 22  (mod 5) (2)

Nhân (1) với (2), vế theo vế ta có: 24k +  (mod 5)  24k + -  (mod 5) Hay 24k + - chia hết cho với k = 0, 1, 2, hay ta đƣợc vô số số dạng 2n – (n  N) chia hết cho

Chú ý: giải toán đồng dƣ, ta thƣờng quan tâm đến a   (mod m) a  (mod m)  an  (mod m)

a  -1 (mod m)  an  (-1)n (mod m) Ví dụ 3: Chứng minh

a) 2015 – chia hết cho 11 b) 230 + 330 chi hết cho 13 c) 555222 + 222555 chia hết cho

Giải

a) 25  - (mod 11) (1); 10  - (mod 11)  105  - (mod 11) (2)

Từ (1) (2) suy 25 105  (mod 11)  205  (mod 11) 205 –  (mod 11) b) 26  - (mod 13)  230  - (mod 13) (3)

33  (mod 13)  330  (mod 13) (4)

(46)

c) 555  (mod 7)  555222  2222 (mod 7) (5)

23  (mod 7)  (23)74  (mod 7)  555222  (mod 7) (6) 222  - (mod 7)  222555  (-2)555 (mod 7)

Lại có (-2)3  - (mod 7)  [(-2)3]185  - (mod 7)  222555  - (mod 7) Ta suy 555222 + 222555  - (mod 7) hay 555222 + 222555 chia hết cho Ví dụ 4: Chứng minh số 24n +

2 + chia hết cho 11 với số tự nhiên n Thật vậy:Ta có: 25  - (mod 11)  210  (mod 11)

Xét số dƣ chia 24n + cho 10 Ta có: 24  (mod 5)  24n  (mod 5)  2.24n  (mod 10)  24n +  (mod 10)  24n + = 10 k +

Nên 24n +

2 + = 210k + + =4 210k + = 4.(BS 11 + 1)k + = 4.(BS 11 + 1k) + = BS 11 + 11 chia hết cho 11

Bài tập nhà:

Bài 1: CMR:

a) 228 – chia hết cho 29

b)Trong số có dạng2n – có vơ số số chia hết cho 13 Bài 2: Tìm số dƣ chia A = 2011 + 2212 + 19962009 cho

CHUYÊN ĐỀ 10 – TÍNH CHIA HẾT ĐỐI VỚI ĐA THỨC

A Dạng 1: Tìm dƣ phép chia mà không thực phép chia

1 Đa thức chia có dạng x – a (a hằng) a) Định lí Bơdu (Bezout, 1730 – 1783):

Số dƣ phép chia đa thức f(x) cho nhị thức x – a giá trị f(x) x = a Ta có: f(x) = (x – a) Q(x) + r

Đẳng thức với x nên với x = a, ta có f(a) = 0.Q(a) + r hay f(a) = r

(47)

20 CHUYÊN ĐỀ BỒI DƢỠNG TỐN b) f(x) có tổng hệ số chia hết cho x –

c) f(x) có tổng hệ số hạng tử bậc chẵn tổng hệ số hạng tử bậc lẻ chia hết cho x +

Ví dụ : Khơng làm phép chia, xét xem A = x3 – 9x2 + 6x + 16 chia hết cho B = x + 1, C = x – không

Kết quả:

A chia hết cho B, không chia hết cho C Đa thức chia có bậc hai trở lên

Cách 1: Tách đa thức bị chia thành tổng đa thức chia hết cho đa thức chia dƣ Cách 2: Xét giá trị riêng: gọi thƣơng phép chia Q(x), dƣ ax + b

f(x) = g(x) Q(x) + ax + b

Ví dụ 1: Tìm dƣ phép chia x7 + x5 + x3 + cho x2 – Cách 1: Ta biết x2n – chia hết cho x2 – nên ta tách: x7 + x5 + x3 + = (x7 – x) + (x5 – x) +(x3 – x) + 3x +

= x(x6 – 1) + x(x4 – 1) + x(x2 – 1) + 3x + chia cho x2 – dƣ 3x + Cách 2:

Gọi thƣơng phép chia Q(x), dƣ ax + b, Ta có: x7 + x5 + x3 + = (x -1)(x + 1).Q(x) + ax + b với x Đẳng thức với x nên với x = 1, ta có = a + b (1) với x = - ta có - = - a + b (2)

Từ (1) (2) suy a = 3, b =1 nên ta đƣợc dƣ 3x + Ghi nhớ:

an – bn chia hết cho a – b (a  -b)

an + bn ( n lẻ) chia hết cho a + b (a  -b) Ví dụ 2: Tìm dƣ phép chia a) x41 chia cho x2 +

b) x27 + x9 + x3 + x cho x2 –

(48)

Giải

a) x41 = x41 – x + x = x(x40 – 1) + x = x[(x4)10 – 1] + x chia cho x4 – dƣ x nên chia cho

x2 + dƣ x

b) x27 + x9 + x3 + x = (x27 – x) + (x9– x) + (x3 – x) + 4x = x(x26 – 1) + x(x8 – 1) + x(x2 – 1) + 4x chia cho x2 – dƣ 4x

c) x99 + x55 + x11 + x + = x(x98 + 1) + x(x54 + 1) + x(x10 + 1) – 2x + chia cho x2 + dƣ – 2x +

B Sơ đồ HORNƠ

1 Sơ đồ

Để tìm kết phép chia f(x) cho x – a (a số), ta sử dụng sơ đồ hornơ

Nếu đa thức bị chia a0x

+ a1x

+ a2x + a3,

đa thức chia x – a ta đƣợc thƣơng

b0x2 + b1x + b2, dƣ r ta có

Ví dụ:

Đa thức bị chia: x3

-5x2 + 8x – 4, đa thức chia x – Ta có sơ đồ

1 - -

2 + (- 5) = -3 2.(- 3) + = r = 2 +(- 4) = Vậy: x3 -5x2 + 8x – = (x – 2)(x2 – 3x + 2) + phép chia hết

2 Áp dụng sơ đồ Hornơ để tính giá trị đa thức x = a Giá trị f(x) x = a số dƣ phép chia f(x) cho x – a Ví dụ 1:

r= ab2 + a3 a3

b2 = ab1+ a2 b1= ab0+ a1

a2 a1

b0 = a0 a0

(49)

20 CHUYÊN ĐỀ BỒI DƢỠNG TỐN Tính giá trị A = x3 + 3x2 – x = 2010

Ta có sơ đồ:

1 -4

a = 2010 2010.1+3 = 2013 2010.2013 + = 4046130

2010.4046130 – = 8132721296 Vậy: A(2010) = 8132721296

C Chƣngs minh đa thức chia hết cho đa thức khác I Phƣơng pháp:

1 Cách 1: Phân tích đa thức bị chia thành nhân tử có thừa số đa thức chia

2 Cách 2: biến đổi đa thức bị chia thành tổng đa thức chia hết cho đa thức chia Cách 3: Biến đổi tƣơng đƣơng f(x)  g(x) f(x)  g(x)  g(x)

4 cách 4: Chứng tỏ nghiệm đa thức chia nghiệm đa thức bị chia

II Ví dụ 1.Ví dụ 1:

Chứng minh rằng: x8n + x4n + chia hết cho x2n + xn +

Ta có: x8n + x4n + = x8n + 2x4n + - x4n = (x4n + 1)2 - x4n = (x4n + x2n + 1)( x4n - x2n + 1) Ta lại có: x4n + x2n + = x4n + 2x2n + – x2n = (x2n + xn + 1)( x2n - xn + 1)

chia hết cho x2n + xn +

Vậy: x8n + x4n + chia hết cho x2n + xn +

2 Ví dụ 2:

Chứng minh rằng: x3m + + x3n + + chia hết cho x2 + x + với m, n  N Ta có: x3m + + x3n + + = x3m + - x + x3n + – x2 + x2 + x +

= x(x3m – 1) + x2(x3n – 1) + (x2 + x + 1)

Vì x3m – x3n – chia hết cho x3 – nên chia hết cho x2 + x + Vậy: x3m + + x3n + + chia hết cho x2 + x + với m, n  N

3 Ví dụ 3: Chứng minh

(50)

= x9(x90 – 1) + x8(x80 – 1) + + x(x10 – 1) chia hết cho x10 – Mà x10 – = (x – 1)(x9 + x8 + x7 + + x + 1) chia hết cho x9 + x8 + x7 + + x + Suy f(x) – g(x) chia hết cho g(x) = x9 + x8 + x7 + + x +

Nên f(x) = x99 + x88 + x77 + + x11 + chia hết cho g(x) = x9 + x8 + x7 + + x +

4 Ví dụ 4: CMR: f(x) = (x2 + x – 1)10 + (x2 - x + 1)10 – chia hết cho g(x) = x2 – x Đa thức g(x) = x2

– x = x(x – 1) có nghiệm x = x =

Ta có f(0) = (-1)10 + 110 – =  x = nghiệm f(x)  f(x) chứa thừa số x

f(1) = (12 + – 1)10 + (12 – + 1)10 – =  x = nghiệm f(x) f(x) chứa thừa số x – 1, mà thừa số x x – khơng có nhân tử chung, f(x) chia hết cho x(x – 1) hay f(x) = (x2 + x – 1)10 + (x2 - x + 1)10 – chia hết cho g(x) = x2 – x

5 Ví dụ 5: Chứng minh

a) A = x2 – x9 – x1945 chia hết cho B = x2 – x + b) C = 8x9 – 9x8 + chia hết cho D = (x – 1)2

c) C (x) = (x + 1)2n – x2n – 2x – chia hết cho D(x) = x(x + 1)(2x + 1) Giải

a) A = x2 – x9 – x1945 = (x2 – x + 1) – (x9 + 1) – (x1945 – x) Ta có: x2 – x + chia hết cho B = x2 – x +

x9 + chia hết cho x3 + nên chia hết cho B = x2 – x +

x1945 – x = x(x1944 – 1) chia hết cho x3 + (cùng có nghiệm x = - 1) nên chia hết cho B = x2 – x +

Vậy A = x2 – x9 – x1945 chia hết cho B = x2 – x +

b) C = 8x9 – 9x8 + = 8x9 – - 9x8 + = 8(x9 – 1) – 9(x8 – 1) = 8(x – 1)(x8 + x7 + + 1) – 9(x – 1)(x7+ x6 + + 1) = (x – 1)(8x8 – x7 – x6 – x5 – x4 – x3 – x2 – x – 1)

(8x8 – x7 – x6 – x5 – x4 – x3 – x2 – x – 1) chia hết cho x – có tổng hệ số suy (x – 1)(8x8 – x7 – x6 – x5 – x4 – x3 – x2 – x – 1) chia hết cho (x – 1)2

c) Đa thức chia D (x) = x(x + 1)(2x + 1) có ba nghiệm x = 0, x = - 1, x = -

(51)

20 CHUN ĐỀ BỒI DƢỠNG TỐN Ta có:

C(0) = (0 + 1)2n – 02n – 2.0 – =  x = nghiệm C(x)

C(-1) = (-1 + 1)2n – (- 1)2n – 2.(- 1) – =  x = - nghiệm C(x) C(-

2) = (-1 + 1)

2n

– (-1

2) 2n

– 2.(-

2) – =  x = -

2 nghiệm C(x)

Mọi nghiệm đa thức chia nghiệm đa thức bị chia đpcm Ví dụ 6:

Cho f(x) đa thức có hệ số nguyên Biết f(0), f(1) số lẻ Chứng minh f(x) khơng có nghiệm ngun

Giả sử x = a nghiệm nguyên f(x) f(x) = (x – a) Q(x) Trong Q(x) đa thức có hệ số nguyên, f(0) = - a Q(0), f(1) = (1 – a) Q(1)

Do f(0) số lẻ nên a số lẻ, f(1) số lẻ nên – a số lẻ, mà – a hiệu số lẻ số lẻ, mâu thuẩn

Vậy f(x) khơng có nghiệm ngun

Bài tập nhà:

Bài 1: Tìm số dƣ a) x43 chia cho x2 +

b) x77 + x55 + x33 + x11 + x + cho x2 +

Bài 2: Tính giá trị đa thức x4 + 3x3 – x = 2009 Bài 3: Chứng minh

(52)

CHUYÊN ĐỀ 11 – CÁC BÀI TOÁN VỀ BIỂU THỨC HỮU TỈ

A Nhắc lại kiến thức:

Các bƣớc rút gọn biểu thức hửu tỉ

a) Tìm ĐKXĐ: Phân tích mẫu thành nhân tử, cho tất nhân tử khác b) Phân tích tử thành nhân , chia tử mẫu cho nhân tử chung

B Bài tập:

Bài 1: Cho biểu thức A =

4

4

5

10

x x

x x

    a) Rút gọn A

b) tìm x để A =

c) Tìm giá trị A 2x 1

Giải a)Đkxđ :

x4 – 10x2 +   [(x2)2 – x2] – (9x2 – 9)   x2(x2 – 1) – 9(x2 – 1) 

(x2 – 1)(x2 – 9)  (x – 1)(x + 1)(x – 3)(x + 3) 

x

x 1

x 3

x x x

 

     

 

   

    

Tử : x4 – 5x2 + = [(x2)2 – x2] – (x2 – 4) = x2(x2 – 1) – 4(x2 – 1) = (x2 – 1)(x2 – 4) = (x – 1)(x + 1)(x – 2)(x + 2)

Với x   1; x  

A = (x - 1)(x + 1)(x - 2)(x + 2) (x - 2)(x + 2)

(x - 1)(x + 1)(x - 3)(x + 3) (x - 3)(x + 3)

b) A =  (x - 2)(x + 2)

(x - 3)(x + 3) =  (x – 2)(x + 2) =  x = 

c) 2x 1 

2

x x x

x x x

   

  

         

  

* Với x = A = (x - 2)(x + 2) (4 - 2)(4 + 2) 12

(53)

20 CHUYÊN ĐỀ BỒI DƢỠNG TỐN * Với x = - A không xác định

2 Bài 2:

Cho biểu thức B =

3

3

2 12 45

3 19 33

x x x

x x x

      a) Rút gọn B

b) Tìm x để B > Giải

a) Phân tích mẫu: 3x3 – 19x2 + 33x – = (3x3 – 9x2) – (10x2 – 30x) + (3x – 9) = (x – 3)(3x2 – 10x + 3) = (x – 3)[(3x2 – 9x) – (x – 3)] = (x – 3)2(3x – 1)

Đkxđ: (x – 3)2

(3x – 1)   x  x 

3

b) Phân tích tử, ta có:

2x3 – 7x2 – 12x + 45 = (2x3 – 6x2 ) - (x2 - 3x) – (15x - 45) = (x – 3)(2x2 – x – 15) = (x – 3)[(2x2 – 6x) + (5x – 15)] = (x – 3)2(2x + 5)

Với x  x 

3

Thì B =

3

3

2 12 45

3 19 33

x x x

x x x

      =

2

(x - 3) (2x + 5) 2x + (x - 3) (3x - 1)  3x -

c) B >  2x +

3x - > 

1

3 5 1

2 2 3

5

3 1

2

2

5

x x

x x

x x

x x

x

x    

    

  

     

 

   

       

     

 

   

  

3 Bài

Cho biểu thức C = 2 :1 22

1 1

x x

x x x x

 

   

     

 

a) Rút gọn biểu thức C

(54)

a) Đkxđ: x  

C = 2 :1 22 2(1 ) ( 1)( 1)

1 1 (1 )(1 ) 2

x x x x x x

x x x x x x x x

 

        

     

           

   

b) B có giá trị nguyên x số nguyên

2x

 có giá trị nguyên

 2x – Ƣ(2) 

2 1

2 1

2 1,

2

x x x x x x x x                           

Đối chiếu Đkxđ có x = thoả mãn

4 Bài

Cho biểu thức D =

3

2

2

2

x x x

x x x

 

  

a) Rút gọn biểu thức D

b) Tìm x nguyên để D có giá trị ngun c) Tìm giá trị D x =

Giải

a) Nếu x + > x2 = x + nên D =

3

2

2

2

x x x

x x x

 

   =

3 2

2

2 ( 1)( 2)

( 2) ( 2) ( 2)( 2)

x x x x x x x x

x x x x x x x

      

       Nếu x + < x2 = - (x + 2) nên

D =

3

2

2

2

x x x

x x x

 

   =

3

2

2 ( 1)( 2)

( 2) ( 2) ( 2)( 2)

x x x x x x x

x x x x x x x

    

 

         Nếu x + =  x = -2 biểu thức D khơng xác định b) Để D có giá trị ngun

2

2

xx

2

x

có giá trị nguyên

+)

2

2

xx

có giá trị nguyên 

2

x(x - 1) x - x

x > - x > -

        

(55)

20 CHUYÊN ĐỀ BỒI DƢỠNG TOÁN +)

2 x

có giá trị nguyên  x x = 2k 2k (k Z; k < - 1) x < - x < - x

 

   

 

 

c) Khia x =  x > - nên D =

2

2

xx

= 6(6 1) 15

 

Bài tập nhà

Bài 1:

Cho biểu thức A = 2 :

3

x x x x

x x x x x

  

      

        

   

a) Rút gọn A

b) Tìm x để A = 0; A > Bài 2:

Cho biểu thức B =

3

3

3

2

y y y

y y y

      a) Rút gọn B

b) Tìm số nguyên y để 2D

2y + có giá trị nguyên

(56)

CHUYÊN ĐỀ 12 – CÁC BÀI TOÁN VỀ BIỂU THỨC (TIẾP)

* Dạng 2: Các biểu thức có tính quy luật Bài 1: Rút gọn biểu thức

a) A =

 2

2

3

(1.2) (2.3) ( 1) n n n

  

Phƣơng pháp: Xuất phát từ hạng tử cuối để tìm quy luật Ta có

 2

2 ( 1)

n n n

 = 2 2

2 1

( 1) ( 1)

n

n n n n

 

  Nên

A = 12 12 12 12 12 12 12 2 1 2 ( 1)2

1 2 3 ( 1) ( 1) ( 1)

n n

n n n n n

           

  

b) B = 12 12 12 12

2 n

           

       

       

Ta có

2

2 2

1 ( 1)( 1)

1 k k k

k k k

  

   Nên

B = 1.3 2.4 3.52 2 2 ( 1)(2 1) 1.3.2.4 (2 2 2 1)(2 1) 1.2.3 ( 1) 3.4.5 ( 1) 1

2 4 2.3.4 ( 1) 2.3.4 2

n n n n n n n n

n n n n n n n

           

c) C = 150 150 150 150 5.88.11 11.14  47.50 =

1 1 1 1

150

3 8 11 47 50

       

 

 

= 50 1 50.9 45

5 50 10

   

 

 

d) D = 1

1.2.32.3.43.4.5 (n1) (n n1) =

1 1 1 1

2 1.2 2.3 2.3 3.4 (n 1)n n n( 1)

 

     

   

 

= 1 ( 1)( 2)

2 1.2 ( 1) ( 1)

n n

n n n n

    

   

 

Bài 2:

a) Cho A =

1 2

m m

m n

 

   

  ; B =

1 1

2  3 n Tính A B

Ta có A =

1

1 1

1 ( 1)

1 2 n 2

n n n n

n n

n nn n

 

                

 

       

(57)

20 CHUYÊN ĐỀ BỒI DƢỠNG TOÁN

= 1 1 1 1 nB

1 2 2

n n

n n n n

           

       

    

A B = n

b) A = 1 1

1.(2n - 1)3.(2n - 3) (2n - 3).3(2n - 1).1 ; B = +

1

3 2n -

Tính A : B Giải

A = 1 1 1 1

2n 2n - 2n - 2n - 3 2n -

           

       

       

 

1 1 1 1

1

2n 2n - 2n - 2n - 2n - 3

1 1 1 A

.2 2.B

2n 2n - 2n - 2n B n

                                   

Bài tập nhà

Rút gọn biểu thức sau: a) 1 + +

1.22.3 (n - 1)n b)

2 2

2 2

1 n

2 1 1 1 (n + 1) 1

c) 1 + + 1.2.32.3.4 n(n + 1)(n +2)

* Dạng 3: Rút gọn; tính giá trị biểu thức thoả mãn điều kiện biến Bài 1: Cho x

x

  Tính giá trị biểu thức sau :

a) A x2 12 x

  ; b) B x3 13 x

  ; c) C x4 14 x

  ; d) D x5 15 x   Lời giải a) 2 1

A x x

x x              ; b) 3

1 1

B x x x 27 18

x x x

                     ; c) 4 1

C x x 49 47

x x

 

        

(58)

d) A.B x2 12 x3 13 x5 x 15 D

x x x x

  

 

         

    D = 7.18 – = 123

Bài 2: Cho x + y + z = a b c (1);

a b c

+ + = x y z (2)

Tính giá trị biểu thức D =

2

2

a b c

+ +

x y z

 

   

 

   

      Từ (1) suy bcx + acy + abz = (3)

Từ (2) suy

2

2 2

a b c ab ac bc a b c ab ac bc

+ + + + +

x y z xy xz yz x y z xy xz yz

       

              

       

       

                (4)

Thay (3) vào (4) ta có D = – 2.0 =

Bài

a) Cho abc = 2; rút gọn biểu thức A = a b 2c

ab + a + 2bc + b + 1ac + 2c +

Ta có :

A = a ab 2c a ab 2c

ab + a + 2abc + ab + aac + 2c + 2ab + a + 22 + ab + aac + 2c + abc

= a ab 2c a ab ab + a +

ab + a + 22 + ab + a c(a + + ab)ab + a + 22 + ab + aa + + abab + a + 

b) Cho a + b + c = 0; rút gọn biểu thức B =

2 2

2 2 2 2 2

a b c

a - b - c b - c - a c - b - a

Từ a + b + c = a = -(b + c)  a2 = b2 + c2 + 2bc  a2 - b2 - c2 = 2bc Tƣơng tự ta có: b2 - a2 - c2 = 2ac ; c2 - b2 - a2 = 2ab (Hốn vị vịng quanh), nên B =

2 2 3

a b c a b c

2bc 2ac 2ab 2abc

 

   (1)

a + b + c =  -a = (b + c)  -a3 = b3 + c3 + 3bc(b + c)  -a3 = b3 + c3 – 3abc  a3 + b3 + c3 = 3abc (2)

Thay (2) vào (1) ta có B =

3 3

a b c 3abc

2abc 2abc

   

(59)

20 CHUYÊN ĐỀ BỒI DƢỠNG TOÁN

c) Cho a, b, c đôi khác thoả mãn: (a + b + c)2 = a2 + b2 + c2 Rút gọn biểu thức C =

2 2

2 2

a b c

+

a + 2bc b + 2acc + 2ab

Từ (a + b + c)2 = a2 + b2 + c2  ab + ac + bc =

 a2 + 2bc = a2 + 2bc – (ab + ac + bc) = a2 – ab + bc – ac = (a – b)(a – c) Tƣơng tự: b2 + ac = (b – a)(b – c) ; c2 + 2ab = (c – a)(c – b)

C =

2 2 2

a b c a b c

+ -

(a - b)(a - c) (b - a)(b - c) (c - a)(c - b) (a - b)(a - c) (a - b)(b - c) (a - c)(b - c)

=

2 2

a (b - c) b (a - c) c (b - c) (a - b)(a - c)(b - c)

-

(a - b)(a - c)(b - c) (a - b)(a - c)(b - c)(a - b)(a - c)(b - c) (a - b)(a - c)(b - c) * Dạng 4: Chứng minh đẳng thức thoả mãn điều kiện biến

1 Bài 1: Cho + + =

a b c (1); 2

1 1

+ + = a b c (2)

Chứng minh rằng: a + b + c = abc

Từ (1) suy 12 + 12 + 12 + + + + + 12 + 12 + 12

a b c ab bc ac ab bc ac a b c

      

     

     

 + + 1 a + b + c

ab bc ac   abc   a + b + c = abc

2 Bài 2: Cho a, b, c ≠ a + b + c ≠ thỏa mãn điều kiện 1 1 a  b c a b c Chứng minh ba số a, b, c có hai số đối

Từ suy :

2009 2009 2009 2009 2009 2009

1 1

a b c a b c

Ta có : 1 1

a  b c a b c 

1 1

0 a  b c a b c 

a b a b

0 ab c(a b c)

 

 

 

a b a b c(a b c) ab

(a b) (a + b)(b + c)(c + a) = b c b c abc(a b c)

c a c a

     

 

    

         

   

   

 

Từ suy : 20091 20091 20091 20091 12009 20091 20091

(60)

2009 20091 2009 2009 12009 2009 20091

a b c a  ( c) c a

 20091 20091 20091 2009 20091 2009

a b c a b c

3 Bài 3: Cho a + b c b + c a b c  a  a b  c (1)

chứng minh : ba số a, b, c tồn hai số

Từ (1)  2 2 2 2

a c + ab + bc = b c + ac + a b  a (b - c) - a(c b ) bc(c - b) = 0

 (c – b)(a2 – ac = ab + bc) =  (c – b)(a – b)( a – c) =  đpcm

4 Bài 4: Cho (a2 – bc)(b – abc) = (b2 – ac)(a – abc); abc  a b Chứng minh rằng: + + = a + b + c1

a b c

Từ GT  a2b – b2c - a3bc + ab2c2 = ab2 – a2c – ab3c + a2bc2

 (a2b – ab2) + (a2c – b2c) = abc2(a – b) + abc(a - b)(a + b)  (a – b)(ab + ac + bc) = abc(a – b)(a + b + c)

 ab + ac + bc = a + b + c

abc 

1 1

+ + = a + b + c a b c

5 Bài 5: Cho a + b + c = x + y + z = a + b + c =

x y z ; Chứng minh rằng: ax

+ by2 + cz2 = Từ x + y + z =  x2 = (y + z)2 ; y2 = (x + z)2 ; z2 = (y + x)2

 ax2 + by2 + cz2 = a(y + z)2 + b(x + z)2 + c (y + x)2 = … = (b + c)x2 + (a + c)y2 + (a + b)z2 + 2(ayz + bxz + cxy) (1) Từ a + b + c =  - a = b + c; - b = a + c; - c = a + b (2) Từ a + b + c =

x y z  ayz + bxz + cxy = (3) Thay (2), (3) vào (1); ta có:

ax2 + by2 + cz2 = -( ax2 + by2 + cz2 )  ax2 + by2 + cz2 =

6 Bài 6: Cho a + b c

b - c c - aa - b ; chứng minh: 2

a b c

+

(b - c) (c - a) (a - b) 

Từ a + b c b - c c - a a - b  

2

a b c b ab + ac - c =

b - c a - c b - a (a - b)(c - a)

(61)

20 CHUYÊN ĐỀ BỒI DƢỠNG TOÁN  a 2 b2 ab + ac - c2

(b - c) (a - b)(c - a)(b - c)

 (1) (Nhân hai vế với

b - c)

Tƣơng tự, ta có:

2

2

b c bc + ba - a (c - a) (a - b)(c - a)(b - c)

 (2) ;

2

2

c a ac + cb - b (a - b) (a - b)(c - a)(b - c)

 (3)

Cộng vế (1), (2) (3) ta có đpcm

7 Bài 7:

Cho a + b + c = 0; chứng minh: a - b + b - c c - a c + a b c a b a - b b - c c - a

    

  

   = (1)

Đặt a - b = x ; b - c ;c - a

c a  y b z

c a b

= ;

a - b x b - c  y c - a  z (1)  x + y + z + +

x y z

 

 

 

Ta có: x + y + z + + y + z + x + z + x + y

x y z x y z

   

   

    (2)

Ta lại có:

2

y + z b - c c - a c b bc + ac - a c c(a - b)(c - a - b) c(c - a - b)

x a b a - b ab a - b ab(a - b) ab

           =  

c 2c - (a + b + c) 2c ab  ab (3)

Tƣơng tự, ta có:

2

x + z 2a

y  bc (4) ;

2

x + y 2b z  ac (5)

Thay (3), (4) (5) vào (2) ta có:

  1

x + y + z + +

x y z

 

 

  +

2 2

2c 2a 2b

ab  bc  ac = + abc(a

3

+ b3 + c3 ) (6) Từ a + b + c =  a3 + b3 + c3 = 3abc (7) ?

Thay (7) vào (6) ta có: x + y + z + +

x y z

 

 

  +

2

abc 3abc = + = Bài tập nhà:

1) cho + +

x y z  ; tính giá trị biểu thức A = 2 yz xz xy

+ +

x y z

HD: A = xyz3 + xyz3 + xyz3

x y z ; vận dụng a + b + c =  a

(62)

2) Cho a3 + b3 + c3 = 3abc ; Tính giá trị biểu thức A = a + b + c +

b c a

   

   

   

3) Cho x + y + z = 0; chứng minh rằng: y z x z x y

x y z

      

4) Cho a + b + c = a2 + b2 + c2 = 1; a b c

(63)

20 CHUYÊN ĐỀ BỒI DƢỠNG TOÁN

CHUYÊN ĐỀ 13 – CÁC BÀI TOÁN VỀ TAM GIÁC ĐỒNG DẠNG

A Kiến thức:

* Tam giác đồng dạng:

a) trƣờng hợp thứ nhất: (c.c.c)

ABC A’B’C’  AB = AC = BC A'B' A'C' B'C'

b) trƣờng hợp thứ nhất: (c.g.c)

ABC A’B’C’  AB = AC A'B' A'C' ;

 

A = A'

c Trƣờng hợp đồng dạng thứ ba (g.g)

ABC A’B’C’  A = A' ; B = B' 

AH; A’H’là hai đƣờng cao tƣơng ứng thì: A'H'

AH = k (Tỉ số đồng dạng);

A'B'C' ABC

S

S = K

B Bài tập áp dụng Bài 1:

Cho ABC cóB = C , AB = cm, BC = 10 cm a)Tính AC

b)Nếu ba cạnh tam giác ba số tự nhiên liên tiếp cạnh bao nhiêu?

Giải Cách 1:

Trên tia đối tia BA lấy điểm E cho:BD = BC

ACD ABC (g.g)  AC AD

AB AC

2

AC AB AD =AB.(AB + BD)

  = AB(AB + BC)

= 8(10 + 8) = 144  AC = 12 cm Cách 2:

Vẽ tia phân giác BE ABC ABE ACB

E

D

C B

(64)

64

2

AB AE BE AE + BE AC

= AC = AB(AB + CB)

AC ABCB AB + CB AB + CB = 8(8 + 10) = 144

 AC = 12 cm

b) Gọi AC = b, AB = a, BC = c từ câu a ta có b2 = a(a + c) (1) Vì b > anên b = a + b = a +

+ Nếu b = a + (a + 1)2= a2 + ac 2a + = ac a(c – 2) = a = 1; b = 2; c = 3(loại)

+ Nếu b = a + a(c – 4) = - Với a = c = (loại) - Với a = c = (loại) - với a = c = ; b = Vậy a = 4; b = 5; c =

Bài 2:

Cho ABC cân A, đƣờng phân giác BD; tính BD biết BC = cm; AC = 20 cm

Giải

Ta có CD = BC

AD AC4  CD = cm BC = cm

Bài toán trở

Bài 3:

Cho ABC cân A O trung điểm BC Một điểm O di động AB, lấy điểm E AC cho

2

OB CE =

BD Chứng minh

a) DBO OCE

b) DOE DBO OCE

c) DO, EO lần lƣợt phân giác góc BDE, CED

d) khoảng cách từ O đến đoạn ED không đổi D di động AB Giải

D

C B

A

2

3

1 H

I

E

D

(65)

20 CHUYÊN ĐỀ BỒI DƢỠNG TOÁN a) Từ

2

OB CE =

BD 

CE OB =

OB BD  

B = C (gt)  DBO OCE b) Từ câu a suy O = E3 2 (1)

Vì B, O ,C thẳng hàng nên   

O + DOE EOC 180  (2) tam giác EOC   

2

E + C EOC 180  (3) Từ (1), (2), (3) suy DOE  B C

DOE DBO có DO = OE

DB OC (Do DBO OCE)

và DO = OE

DB OB (Do OC = OB)

  

DOE B C nên DOE DBO OCE

c) Từ câu b suy D = D1 2 DO phân giác góc BDE

Củng từ câu b suy E = E1 2 EO phân giác góc CED

c) Gọi OH, OI khoảng cách từ O đến DE, CE OH = OI, mà O cố định nên OH không đổi OI không đổi D di động AB

Bài 4: (Đề HSG huyện Lộc hà – năm 2007 – 2008)

Cho ABC cân A, có BC = 2a, M trung điểm BC, lấy D, E thuộc AB, AC cho

 

DME = B

a) Chứng minh tích BD CE khơng đổi b)Chứng minh DM tia phân giác BDE

c) Tính chu vi AED ABC tam giác Giải

a) Ta có DMC = DME + CME = B + BDM    , mà DME = B (gt) nên CME = BDM , kết hợp với B = C  (ABC cân A) suy BDM CME (g.g)

 BD BM

= BD CE = BM CM = a

CM CE  không đổi

(66)

20 CHUYÊN ĐỀ BỒI DƢỠNG TOÁN (do BM = CM) DME DBM (c.g.c)  MDE = BMD  hay DM tia phân giác BDE

c) chứng minh tƣơng tự ta có EM tia phân giác DEC

kẻ MH CE ,MI DE, MK DB MH = MI = MK 

DKM = DIM

DK =DI  EIM = EHM EI = EH

Chu vi AED PAED = AD + DE + EA = AK +AH = 2AH (Vì AH = AK)

ABC tam giác nên suy CME củng tam giác CH = MC

2 a

 AH = 1,5a  PAED = AH = 1,5 a = 3a Bài 5:

Cho tam giác ABC, trung tuyến AM Qua điểm D thuộc cạnh BC, vẽ đƣờng thẳng song song với AM, cắt AB, AC E F a) chứng minh DE + DF không đổi D di động BC b) Qua A vẽ đƣờng thẳng song song với BC, cắt FE K Chứng minh K trung điểm FE

Giải

a) DE // AM  DE = BD DE = BD.AM

AM BM BM (1)

DF // AM  DF = CD DF = CD.AM = CD.AM

AM CM CM BM (2)

Từ (1) (2) suy

DE + DF = BD.AM + CD.AM

BM BM =

BD CD BC

+ AM = AM = 2AM

BM BM BM

 

 

  không đổi

b) AK // BC suy FKA AMC (g.g)  FK = KA AM CM (3)

EK KA EK KA EK KA EK KA EK KA

= = =

ED BD ED + EK BD + KAKD BD + DMAM BM AMCM (2)

(Vì CM = BM)

K H

I

M E

D

C B

K F

E

D M

C B

(67)

20 CHUYÊN ĐỀ BỒI DƢỠNG TOÁN Từ (1) (2) suy FK EK

AM AM FK = EK hay K trung điểm FE Bài 6: (Đề HSG huyện Thạch hà năm 2003 – 2004)

Cho hình thoi ABCD cạnh a có 

A = 60 , đƣờng thẳng qua C cắt tia đối tia BA, DA M, N

a) Chứng minh tích BM DN có giá trị khơng đổi

b) Gọi K giao điểm BN DM Tính số đo góc BKD Giải

a) BC // AN  MB = CM BA CN (1)

CD// AM  CM = AD CN DN (2)

Từ (1) (2) suy

2

MB AD

= MB.DN = BA.AD = a.a = a

BA DN

b) MBD vàBDN có MBD = BDN   = 1200

MB MB CM AD BD

= =

BD BA  CN DN DN(Do ABCD hình thoi có

A = 60 nên AB = BC = CD = DA)

MBD BDN

Suy M = B1 1 MBD vàBKD có BDM = BDK  M = B1 1 nên BKD = MBD = 120 

Bài 7:

Cho hình bình hành ABCD có đƣờng chéo lớn AC,tia Dx cắt SC, AB, BC lần lƣợt I, M, N Vẽ CE vng góc với AB, CF vng góc với AD, BG vng góc với AC Gọi K điểm đối xứng với D qua I Chứng minh

a) IM IN = ID2 b) KM = DM

KN DN

c) AB AE + AD AF = AC2 Giải

1

1 K M

N D

C B

A

I

K F

G

E M D

C

B

(68)

a) Từ AD // CM  IM = CI ID AI (1)

Từ CD // AN  CI ID

AIIN (2)

Từ (1) (2) suy IM

ID = ID

IN hay ID

= IM IN

b) Ta có DM = CM DM = CM DM = CM MN MBMN + DM MB + CM  DN CB (3)

Từ ID = IK ID2 = IM IN suy IK2 = IM IN

 IK = IN IK - IM = IN - IK KM = KN KM = IM IM IK IM IK  IM IK  KN IK 

KM IM CM CM

=

KN ID  AD  CB (4)

Từ (3) (4) suy KM = DM

KN DN

c) Ta có AGB AEC  AE = AC AB.AE = AC.AG

AG AB AB AE = AG(AG + CG) (5)

CGB AFC  AF = CG CG

AC CB AD(vì CB = AD)

AF AD = AC CG  AF AD = (AG + CG) CG (6)

Cộng (5) (6) vế theo vế ta có: AB AE + AF AD = (AG + CG) AG + (AG + CG) CG  AB AE + AF AD = AG2 +2.AG.CG + CG2 = (AG + CG)2 = AC2

Vậy: AB AE + AD AF = AC2

Bài tập nhà

Bài

Cho Hình bình hành ABCD, đƣờng thẳng cắt AB, AD, AC lần lƣợt E, F, G Chứng minh: AB + AD = AC

AE AF AG

HD: Kẻ DM // FE, BN // FE (M, N thuộc AC) Bài 2:

Qua đỉnh C hình bình hành ABCD, kẻ đƣờng thẳng cắt BD, AB, AD E, G, F chứng minh:

a) DE2 = FE

(69)

20 CHUYÊN ĐỀ BỒI DƢỠNG TOÁN b) CE2 = FE GE

(Gợi ý: Xét tam giác DFE BCE, DEC BEG) Bài

Cho tam giác ABC vuông A, đƣờng cao AH, trung tuyến BM, phân giác CD cắt điểm Chứng minh

a) BH CM AD HC MA BD 

(70)

CHUYÊN ĐỀ 14 – PHƢƠNG TRÌNH BẬC CAO A.Mục tiêu:

* Củng cố, ôn tập kiến thức kỹ giải Pt bậc cao cách phân tích thành nhân tử

* Khắc sâu kỹ phân tích đa thức thành nhân tử kỹ giải Pt

B Kiến thức tập: I Phƣơng pháp:

* Cách 1: Để giải Pt bậc cao, ta biến đổi, rút gọn để dƣa Pt dạng Pt có vế trái đa thức bậc cao, vế phải 0, vận dụng phƣơng pháp phân tích đa thức thành nhân tử để đƣa Pt dạng pt tích để giải

* Cách 2: Đặt ẩn phụ

II Các ví dụ: 1.Ví dụ 1: Giải Pt

a) (x + 1)2(x + 2) + (x – 1)2(x – 2) = 12

 2x3 + 10x = 12 x3 + 5x – = (x3 – 1) + (5x – 5) (x – 1)(x2 + x + 6) =

2

x = x - =

x

1 23

x + x + = x +

2

 

  

    

  

 

(Vì

2

1 23

x +

2

   

 

  vô nghiệm)

b) x4 + x2 + 6x – = (1)

Vế phải Pt đa thức có tổng hệ số 0, nên có nghiệm x = nên có nhân tử x – 1, ta có

(1)  (x4 – x3) + (x3 – x2) + (2x2 – 2x) + (8x – 8) =

  (x – 1)(x3 + x2 + 2x + 8) (x – 1)[(x3 + 2x2) – (x2 + 2x) + (4x – 8) ] =  (x – 1)[x2(x + 2) – x(x + 2) + 4(x + 2) = (x – 1)(x + 2)(x2 – x + 4) = c) (x – 1)3 + (2x + 3)3 = 27x3 +

(71)

20 CHUYÊN ĐỀ BỒI DƢỠNG TOÁN (2) (6x3 – 18x2) + (7x2 – 21x) + (2x – 6) =

6x2(x – 3) + 7x(x – 3) + 2(x – 3) =  (x – 3)(6x2 + 7x + 2) =

(x – 3)[(6x2 + 3x) + (4x + 2)] =  (x – 3)[3x(2x + 1) + 2(2x + 1)] =  (x – 3)(2x + 1)(3x + 2)

d) (x2 + 5x)2 – 2(x2 + 5x) = 24 [(x2 + 5x)2 – 2(x2 + 5x) + 1] – 25 = (x2 + 5x - 1)2 – 25 = (x2 + 5x - + 5)( (x2 + 5x - – 5) =

(x2 + 5x + 4) (x2 + 5x – 6) = [(x2 + x) +(4x + 4)][(x2 – x) + (6x – 6)] = (x + 1)(x + 4)(x – 1)(x + 6) =

e) (x2 + x + 1)2 = 3(x4 + x2 + 1)  (x2 + x + 1)2 - 3(x4 + x2 + 1) =  (x2 + x + 1)2 – 3(x2 + x + 1)( x2 - x + 1) =

 ( x2 + x + 1)[ x2 + x + – 3(x2 - x + 1)] = ( x2 + x + 1)( -2x2 + 4x - 2) =  (x2 + x + 1)(x2 – 2x + 1) = ( x2 + x + 1)(x – 1)2 =

f) x5 = x4 + x3 + x2 + x +  (x5 – 1) – (x4 + x3 + x2 + x + 1) =  (x – 1) (x4 + x3 + x2 + x + 1) – (x4 + x3 + x2 + x + 1) =

 (x – 2) (x4 + x3 + x2 + x + 1) = +) x – =  x =

+) x4 + x3 + x2 + x + =  (x4 + x3) + (x + 1) + x2 = (x + 1)(x3 + 1) + x2 =  (x + 1)2(x2 – x + 1) + x2 =  (x + 1)2 [(x2 – 2.x.1

2 + 4) +

3 4] + x

2

=

 (x + 1)2

2

1

x + +

2

  

  

 

 

  + x

2

= Vơ nghiệm (x + 1)2

2

1

x + +

2

  

  

 

 

   nhƣng

không xẩy dấu

Bài 2:

a) (x2 + x - 2)( x2 + x – 3) = 12  (x2 + x – 2)[( x2 + x – 2) – 1] – 12 =  (x2 + x – 2)2 – (x2 + x – 2) – 12 =

Đặt x2

+ x – = y Thì

(72)

(x + 3)(x – 2) =

* y + =  x2 + x – + =  x2 + x + = (vô nghiệm)

b) (x – 4)( x – 5)( x – 6)( x – 7) = 1680 (x2 – 11x + 28)( x2 – 11x + 30) = 1680 Đặt x2

– 11x + 29 = y , ta có:

(x2 – 11x + 28)( x2 – 11x + 30) = 1680 (y + 1)(y – 1) = 1680 y2 = 1681 y =  41 y = 41  x2 – 11x + 29 = 41  x2 – 11x – 12 = (x2 – x) + (12x – 12) =

 (x – 1)(x + 12) =

* y = - 41  x2 – 11x + 29 = - 41  x2 – 11x + 70 = (x2 – 2x 11

2 + 121

4 )+ 159

4 =

c) (x2 – 6x + 9)2 – 15(x2 – 6x + 10) = (3) Đặt x2

– 6x + = (x – 3)2 = y  0, ta có

(3) y2 – 15(y + 1) – = y2 – 15y – 16 = (y + 1)(y – 15) = Với y + =  y = -1 (loại)

Với y – 15 = y = 15  (x – 3)2 = 16  x – =  + x – =  x =

+ x – = - x = -

d) (x2 + 1)2 + 3x(x2 + 1) + 2x2 = (4) Đặt x2

+ = y

(4)  y2 + 3xy + 2x2 = (y2 + xy) + (2xy + 2x2) = (y + x)(y + 2x) = +) x + y = x2 + x + = : Vô nghiệm

+) y + 2x =  x2 + 2x + =  (x + 1)2 =  x = -

Bài 3:

a) (2x + 1)(x + 1)2(2x + 3) = 18  (2x + 1)(2x + 2)2(2x + 3) = 72 (1) Đặt 2x + = y, ta có

(1)  (y – 1)y2(y + 1) = 72 y2(y2 – 1) = 72  y4 – y2 – 72 =

Đặt y2

(73)

20 CHUYÊN ĐỀ BỒI DƢỠNG TOÁN * z – =  z = y2 =  y =  x =

b) (x + 1)4 + (x – 3)4 = 82 (2)

Đặt y = x – x + = y + 2; x – = y – 2, ta có (2) (y + 2)4 + (y – 2)4 = 82

y4 +8y3 + 24y2 + 32y + 16 + y4 - 8y3 + 24y2 - 32y + 16 = 82  2y4 + 48y2 + 32 – 82 =  y4 + 24y2 – 25 =

Đặt y2

= z   y4 + 24y2 – 25 =  z2 + 24 z – 25 =  (z – 1)(z + 25) = +) z – =  z = y = 1 x = 0; x =

+) z + 25 = z = - 25 (loại)

Chú ý: Khi giải Pt bậc dạng (x + a)4 + (x + b)4 = c ta thƣờng đặt ẩn phụ y = x + a + b

2

c) (4 – x)5 + (x – 2)5 = 32 (x – 2)5 – (x – 4)5 = 32 Đặt y = x – x – = y + 1; x – = y – 1; ta có: (x – 2)5 – (x – 4)5 = 32  (y + 1)5 - (y – 1)5 = 32

y5 + 5y4 + 10y3 + 10y2 + 5y + – (y5 - 5y4 + 10y3 - 10y2 + 5y - 1) – 32 = 10y4 + 20y2 – 30 =  y4 + 2y2 – =

Đặt y2

= z   y4 + 2y2 – = z2 + 2z – = (z – 1)(z + 3) = d) (x - 7)4 + (x – 8)4 = (15 – 2x)4

Đặt x – = a; x – = b ; 15 – 2x = c - c = 2x – 15  a + b = - c , Nên

(x - 7)4 + (x – 8)4 = (15 – 2x)4 a4 + b4 = c4  a4 + b4 - c4 =  a4 + b4 – (a + b)4 =  4ab(a2 +

2ab + b

) = 

2

2

3

4ab a + b + b

4 16

  

  

 

 

  =  4ab =

(Vì

2

2

3

a + b + b

4 16

 

 

   nhƣng không xẩy dấu bằng)  ab = x = 7; x =

e) 6x4 + 7x3 – 36x2 – 7x + =  2

1

6 x x - 36

x x

     

   

   

(Vì x = không nghiệm) Đặt x -

x = y 

2

1 x

x

(74)

2

1

6 x x - 36

x x

     

   

     6(y

2

+ 2) + 7y – 36 = 6y2 + 7y – 24 = (6y2 – 9y) + (16y – 24) =  (3y + )(2y – 3) =

+) 3y + = y = -

3  x -

x = -

3  (x + 3)(3x – 1) = 0

x = - x + =

1 3x - = x =

3

 

 

 

+) 2y – = y =

2  x -

x =

2  (2x + 1)(x – 2) = 0

x = x - =

1 2x + = x = -

2

 

 

 

Bài 4: Chứng minh rằng: Pt sau vô nghiệm

a) x4 – 3x2 + 6x + 13 = ( x4 – 4x2 + 4) +(x2 + 6x + 9) =  (x2 – 2)2 + (x + 3)2 = Vế trái (x2 – 2)2 + (x + 3)2  nhƣng không đồng thời xẩy x2 = x = -3

b) x6 + x5 + x4 + x3 + x2 + x + =  (x – 1)( x6 + x5 + x4 + x3 + x2 + x + 1) = x7 – =  x =

x = không nghiệm Pt x6 + x5 + x4 + x3 + x2 + x + =

Bài tập nhà: Bài 1: Giải Pt a)(x2 + 1)2 = 4(2x – 1)

HD: Chuyển vế, triển khai (x2 + 1)2, phân tích thành nhân tử: (x – 1)2(x2 + 2x + 5) = b) x(x + 1)(x + 2)(x + 3) = 24 (Nhân nhân tử với nhau, áp dụng PP đặt ẩn phụ) c) (12x + 7)2(3x + 2)(2x + 1) = (Nhân vế với 24, đặt 12x + = y)

d) (x2 – 9)2 = 12x + (Thêm, bớt 36x2)

e) (x – 1)4 + (x – 2)4 = ( Đặt y = x – 1,5; Đs: x = 1; x = 2) f) (x – 1)5 + (x + 3)5 = 242(x + 1) (Đặt x + = y; Đs:0; -1; -2 ) g) (x + 1)3 + (x - 2)3 = (2x – 1)3

Đặt x + = a; x – = b; - 2x = c a + b + c = a3 + b3 + c3 = 3abc h) 6x4 + 5x3 – 38x2 + 5x + = (Chia vế cho x2; Đặt y = x +

x )

(75)

20 CHUYÊN ĐỀ BỒI DƢỠNG TOÁN

Bài 2: Chứng minh pt sau vô nghiệm a) 2x4 – 10x2 + 17 =

(Phân tích vế trái thành tổng hai bình phƣơng) b) x4 – 2x3+ 4x2 – 3x + =

(76)

CHUYÊN ĐỀ – SỬ DỤNG CƠNG THỨC DIỆN TÍCH ĐỂ THIẾT LẬP QUAN HỆ ĐỘ DÀI CỦA CÁC ĐOẠN THẲNG

Ngày soạn:23 – - 2010

A Một số kiến thức:

1 Cơng thức tính diện tích tam giác: S =

2 a.h (a – độ dài cạnh, h – độ dài đƣờng cao tƣơng ứng)

2 Một số tính chất:

Hai tam giác có chung cạnh, có độ dài đƣờng cao có diện tích Hai tam giác có diện tích

B Một số toán: 1 Bài 1:

Cho ABC có AC = 6cm; AB = cm; đƣờng cao AH; BK; CI Biết AH = CI + BK

2

Tính BC Giải

Ta có: BK = 2SABC

AC ; CI =

ABC

2S AB

 BK + CI = SABC

1

AC AB

  

 

 

 2AH = 2.1

2 BC AH

1

AC AB

  

 

  BC

1

AC AB

  

 

  =

 BC = : 1

AC AB

  

 

  = :

1

  

 

  = 4,8 cm

Bài 2:

Cho ABC có độ dài cạnh a, b, c; độ dài đƣờng cao tƣơng ứng ha, hb, hc Biết

rằng a + = b + hb = c + hc Chứng minh ABC tam giác

Giải

Gọi SABC = S

K I

H C

(77)

20 CHUYÊN ĐỀ BỒI DƢỠNG TOÁN Ta xét a + = b + hb  a – b = – hb =

2S 2S 1 a - b

- 2S - 2S

b a b a ab

 

  

 

 a – b = 2S a - b

ab  (a – b)

2S -

ab

 

 

  =  ABC cân C vuông C (1)

Tƣơng tự ta có: ABC cân A vuông A (2); ABC cân B vuông B (3) Từ (1), (2) (3) suy ABC cân vuông ba đỉnh (Không xẩy vuông ba đỉnh)  ABC tam giác

Bài 3:

Cho điểm O nằm tam giác ABC, tia AO, BO, Co cắt cạnh tam giác ABC theo thứ tự A’, B’, C’ Chứng minh rằng:

a) OA' OB' OC'

AA'BB'CC' b)

OA OB OC AA'BB'CC'

c) M = OA OB OC

OA'OB'OC' Tìm vị trí O để tổng M có giá trị nhỏ

d) N = OA OB OC

OA' OB' OC' Tìm vị trí O để tích N có giá

trị nhỏ Giải

Gọi SABC = S, S1 = SBOC , S2 = SCOA , S3 = SAOB Ta có:

3

OA'C OA'B

S S S

S OA

= =

OA' S S S

 (1)

OA'C OA'B OA'C OA'B

AA'C AA'B AA'C AA'B

S S S S S

OA'

= =

AA' S S S S S

 

 (2)

Từ (1) (2) suy OA S2 S3

AA' S

 

Tƣơng tự ta có

S S OB

OB' S

 ;

3

S S OC

OC' S

 ; OB' S2

BB'  S ;

3

S OC' CC'  S

a) OA' OB' OC' S1 S2 S3 S 1

AA'BB'CC' S  S  S  S

b) OA OB OC S2 S3 S1 S3 S1 S2 2S

2

AA' BB' CC' S S S S

  

      

C' B'

A' O

C B

(78)

c) M = 3 2 3

1 2 3

S S S S S S S S S S S S

OA OB OC

OA' OB' OC' S S S S S S S S S

   

 

  

            

     

Aùp dụng Bđt Cơ si ta có 3

2 3

S S

S S S S

2 2

S S S S S S

   

 

         

 

     

Đẳng thức xẩy S1 = S2 = S3  O trọng tâm tam giác ABC

d) N = 3  3 3 2

1 3

S S S S S S S S S S S S

S S S S S S

  

   

 N2 =      

   

2 2

2 3 2 3

2

1 3

S S S S S S 4S S 4S S 4S S 64

S S S S S S

  

   N  Đẳng thức xẩy S1 = S2 = S3  O trọng tâm tam giác ABC Bài 4:

Cho tam giác ABC, đƣờng caoAD, BE, CF; gọi A’, B’, C’ hình chiếu M (nằm bên tam giác ABC) AD, BE, CF Chứng minh rằng: Khi M thay đổi vị trí tam giác ABC thì:

a) A’D + B’E + C’F không đổi b) AA’ + BB’ + CC’ không đổi Giải

Gọi h = AH chiều cao tam giác ABC h khơng đổi

Gọi khoảng cách từ M đến cạnh AB; BC; CA MP; MQ; MR A’D + B’E + C’F = MQ + MR + MP

Vì M nằm tam giác ABC nên SBMC + SCMA + SBMA = SABC

 BC.(MQ + MR + MP) = BC AH

 MQ + MR + MP = AH  A’D + B’E + C’F = AH = h Vậy: A’D + B’E + C’F = AH = h không đổi

b) AA’ + BB’ + CC’ = (AH – A’D)+(BE – B’E) (CF – C’F) = (AH + BE + CF) – (A’D + B’E + C’F) = 3h – h = 2h không đổi

Bài 5:

R

Q P

C'

B' A' M

F E

D C

B

(79)

20 CHUYÊN ĐỀ BỒI DƢỠNG TỐN

Cho tam giác ABC có BC trung bình cộng AC AB; Gọi I giao điểm phân giác, G trọng tâm tam giác Chứng minh: IG // BC

Giải

Gọi khoảng cách từ a, I, G đến BC lần lƣợt AH, IK, GD Vì I giap điểm ba đƣờng phân giác nên khoảng cách từ I đến ba cạnh AB, BC, CA IK

Vì I nằm tam giác ABC nên:

SABC = SAIB + SBIC + SCIA BC.AH = IK(AB+BC+CA) (1)

Mà BC = AB + CA

2  AB + CA = BC (2)

Thay (2) vào (1) ta có: BC AH = IK 3BC  IK =

3AH (a)

Vì G trọng tâm tam giác ABC nên: SBGC =

1

3 SABC  BC GD =

3 BC AH  GD =

3 AH (b)

Từ (a) (b) suy IK = GD hay khoảng cách từ I, G đến BC nên IG // BC

Bài tập nhà:

1) Cho C điểm thuộc tia phân giác 

xOy = 60 , Mlà điểm nằm đƣờng vng góc với OC C thuộc miền xOy, gọi MA, MB thứ tự khoảng cách từ M đến Ox, Oy Tính độ dài OC theo MA, MB

2) Cho M điểm nằm tam giác ABC A’, B’, C’ hình chiếu M cạnh BC, AC, AB Các đƣờng thẳng vng góc với BC C, vng góc với CA A , vng góc với AB B cắt D, E, F Chứng minh rằng:

a) Tam giác DEF tam giác

b) AB’ + BC’ + CA’ không phụ thuộc vị trí M tam giác ABC

M K

H

G I

D C

(80)

CHUYÊN ĐỀ 16 – BẤT ĐẲNG THỨC

Phần I : kiến thức cần lƣu ý

1-Đinhnghĩa:

0

A B A B

A B A B

    

    

2-tính chất + A>B BA

+ A>B B >C  A > C + A>B  A + C >B + C

+ A>B C > D  A +C > B + D + A>B C >  A.C > B.C + A>B C <  A.C < B.C

+ < A < B < C < D  < A.C < B.D

+ A > B >  An > Bn n

+ A > B  An > Bn với n lẻ + A > B  An > Bn với n chẵn + m > n > A >  Am

> An + m > n > <A <  Am

< An +A < B A.B > 

B A

1

3 - số bất đẳng thức + A2 

với A ( dấu = xảy A = ) + An  vớiA ( dấu = xảy A = ) + A 0 với A (dấu = xảy A = ) + - A < A = A

+ ABAB ( dấu = xảy A.B > 0) + ABAB ( dấu = xảy A.B < 0)

Phần II : số phƣơng pháp chứng minh bất đẳng thức 1) Phƣơng pháp 1: dùng định nghĩa

Kiến thức : Để chứng minh A > B Ta chứng minh A – B > Lƣu ý dùng bất đẳng thức M2 

với  M

Ví dụ  x, y, z chứng minh : a) x2

+ y2

+ z2 

xy+ yz + zx b) x2

+ y2

+ z2 

(81)

20 CHUYÊN ĐỀ BỒI DƢỠNG TOÁN Giải:

a) Ta xét hiệu : x2

+ y2

+ z2

- xy – yz – zx =

2

.2 ( x2

+ y2

+ z2

- xy – yz – zx) =

2

1 2

(x y) (x z) (y z)

      

   với x;y;zR

Vì (x-y)2 0 vớix ; y Dấu xảy x = y (x- z)2 0 vớix ; z Dấu xảy x = z (y- z)2 0 với z; y Dấu xảy z = y

Vậy x2 + y2 + z2  xy+ yz + zx Dấu xảy x = y =z

b)Ta xét hiệu: x2

+ y2

+ z2

- ( 2xy – 2xz +2yz ) = x2

+ y2

+ z2

- 2xy +2xz –2yz = ( x – y + z)2

0

 với x;y;zR

Vậy x2

+ y2

+ z2 

2xy – 2xz + 2yz với x;y;zR

Dấu xảy x + y = z

Ví dụ 2: chứng minh : a)

2

2

2

2 

     

b a b

a

; b)

2

2

3

3 

       

b c a b c a

c) Hãy tổng quát toán giải

a) Ta xét hiệu

2

2

2

2 

     

b a b

a

=  

4

2a2 b2 a2  abb2

= 2a 2b a b 2ab

1    

=  

1  

b a Vậy 2 2

2 

     

b a b

a

Dấu xảy a = b

b)Ta xét hiệu:

2

2

3

3 

       

b c a b c

a

=       

1      

a c c b b a Vậy 2 2

3 

       

b c a b c a

Dấu xảy a = b =c

c)Tổng quát: 2 2 2

1 

            n a a a n a a

(82)

* Tóm lại bƣớc để chứng minh AB theo định nghĩa Bƣớc 1: Ta xét hiệu H = A - B

Bƣớc 2:Biến đổi H = (C+D)2hoặc H=(C+D)2+….+(E+F)2

Bƣớc 3: Kết luận A  B

2) phƣơng pháp : Dùng phép biến đổi tƣơng đƣơng

Lƣu ý:

Ta biến đổi bất đẳng thức cần chứng minh tƣơng đƣơng với bất đẳng thức bất đẳng thức đƣợc chứng minh

Ví dụ 1: Cho a, b, c, d,e số thực chứng minh a) abab

4

2

b)a2b21abab c)a2b2c2d2e2 abcde

Giải:

a) abab

4 2 ab b a

4 2 

 4a2 4ab2 0 2ab2 0 (Bđt đúng)

Vậyabab

4

2

2 (dấu xảy 2a = b)

b) a2b21abab2(a2b21 2(abab)

1

2

2 2

2        

a ab b a a b b (ab)2(a1)2(b1)2 0 (luôn đúng) Vậy a2b21ababDấu xảy a = b =

c) a2b2c2d2 e2 abcde 4 a2b2c2d2e2 4abcde

 a24ab4b2  a24ac4c2  a24ad4d2  a24ac4c20

 a2b 2 a2c 2 a2d 2 a2c2 0

Ví dụ 2: Chứng minh rằng:  10 10 2  8 4

b a b a b a b

a     

Giải:

 10 10 2  8 4 b a b a b a b

a       12 10 2 10 12 12 4 12

b b a b a a b b a b a

a       

a8b2a2 b2a2b8b2a20  a2b2(a2-b2)(a6-b6)  a2b2(a2-b2)2(a4+ a2b2+b4)  Ví dụ 4: cho ba số thực khác không x, y, z thỏa mãn:

(83)

20 CHUYÊN ĐỀ BỒI DƢỠNG TOÁN Chứng minh : có ba số x,y,z lớn Giải: Xét (x-1)(y-1)(z-1) = xyz + (xy + yz + zx) + x + y + z - = (xyz - 1) + (x + y + z) - xyz(

z y x 1 

 ) = x + y + z - (1 1) 

z y x (vì z y x 1 1 

< x+y+z theo gt)  số x-1 , y-1 , z-1 âm ba sỗ-1 , y-1, z-1 dƣơng

Nếủ trƣờng hợp sau xảy x, y, z >1 x.y.z>1 Mâu thuẫn gt x.y.z =1 bắt buộc phải xảy trƣờng hợp tức có ba số x ,y ,z số lớn

3) Phƣơng pháp 3: dùng bất đẳng thức quen thuộc A) số bất đẳng thức hay dùng

1) Các bất đẳng thức phụ: a) x2 y2 2xy b)

xy y

x2   dấu( = ) x = y = c) xy2 4xy d)  2

a b b a

2)Bất đẳng thức Cô sy: n n aa a an n a a a a 3

1      Với 0

i a

3)Bất đẳng thức Bunhiacopski

    2 2 1 2 2 2 2

2 a an x x n a x a x anxn

a          

4) Bất đẳng thức Trê-bƣ - sép: Nếu        C B A c b a  3 C B A c b a cC bB

aA      

Nếu        C B A c b a  3 C B A c b a cC bB

aA    

  

Dấu xảy

       C B A c b a

B) ví dụ

ví dụ

(84)

Giải: Dùng bất đẳng thức phụ: xy2 4xy

Tacó ab2 4ab; bc2 4bc ; ca2 4ac

 2

b

a bc2 ca2 64a2b2c2 8abc2(a + b)(b + c)(c + a)  8abc Dấu “=” xảy a = b = c

ví dụ 2: Cho a > b > c > a2b2c2 1 chứng minh

3 3

1

a b c

b c a c a b 

Do a,b,c đối xứng , giả sử a  b  c 

           b a c c a b c b

a a b c

2 2

áp dụng BĐT Trê- bƣ-sép ta có

                 

a b

c c a b c b a c b a b a c c c a b b c b a a 2 2 2 = = Vậy 3     

a b

c c a b c b a

Dấu xảy a = b = c =

3

ví dụ 3: Cho a,b,c,d > abcd =1 .Chứng minh :

      10

2 2

2         

a c d d c b c b a d c b a

Ta có a2b2 2ab; c2d2 2cd

Do abcd =1 nên cd = ab (dùng 1   x x )

Ta có   2(  )2(  )4 ab ab cd ab c b

a (1) Mặt khác: abc bcd d ca = (ab + cd) + (ac + bd) + (bc + ad)

= 1 222

                      bc bc ac ac ab

aba2b2c2d2 abc b cd d ca10

ví dụ 4: Chứng minh : a2b2c2 abbcac

Giải: Dùng bất đẳng thức Bunhiacopski

Xét cặp số (1,1,1) (a,b,c) ta có  2 2 2  2

) ( 1

1   abcabc

 3a2 b2 c2a2b2c22abbcac a2b2 c2 abbcac (đpcm) Dấu xảy a = b = c

(85)

20 CHUYÊN ĐỀ BỒI DƢỠNG TOÁN

A Kiến thức

1) Cho a, b ,c số dƣơng a – Nếu 1

b a c b c a b a  

 b – Nếu 1

b a c b c a b a   

2) Nếu b,d >0 từ

d c d b c a b a d c b a      

B Các ví dụ:

ví dụ 1: Cho a,b,c,d > Chứng minh :1 2

            b a d d a d c c d c b b c b a a

Theo tính chất tỉ lệ thức ta có

d c b a d a c b a a c b a a          

 (1)

Mặt khác :

d c b a a c b a a     

 (2)

Từ (1) (2) ta có

d c b a a  

 < a b c a

 <a b c d

d a     (3) Tƣơng tự ta có :

d c b a a b d c b b d c b a b          

 (4)

d c b a c b a d c c d c b a c          

 (5); a b c d

c d b a d d d c b a d          

 (6)

cộng vế với vế (3); (4); (5); (6) ta có

2              b a d d a d c c d c b b c b a a (đpcm) ví dụ : Cho:

b a

<

d c

b,d > Chứng minh

b a < d c d b cd ab    2

Giải: Từ b a < d c 2 d cd b ab    d c d cd d b cd ab b

ab  

 

 2 2 2

2  b a < d c d b cd ab    2 (đpcm)

ví dụ : Cho a;b;c;d số nguyên dƣơng thỏa mãn : a + b = c+d =1000 tìm giá trị lớn

d b c a

giải : Khơng tính tổng qt ta giả sử :

c a d bd b d c b a c a    

 ; 1

c a

(86)

a, Nếu: b 998 d b 998   d b c a

  999

b, Nếu: b = 998 a =1 

d b c a = d c 999 1

Đạt giá trị lớn d = 1; c = 999 Vậy: giá trị lớn

d b c a

= 999 +

999

a = d = 1; c = b = 999 Ví dụ : Với số tự nhiên n >1 chứng minh :

4 1         n n n n Ta có n n n k n 1   

 với k = 1,2,3,…,n-1

Do đó: 2 2 1           n n n n n n n

Ví dụ 5: CMR: A = 2 2 2 12 1 n    

 với n ≥ không số tự nhiên

HD: 12 ; 12 ; 1.2 2.3

Ví dụ 6: Cho a ,b ,c ,d > Chứng minh :

a b b c c d d a a b c b c d c d a d a b

   

    

       

Giải :

Vì a ,b ,c ,d > nên ta có: a b a b a b d

a b c d a b c a b c d

   

 

        (1)

b c b c b c a

a b c d b c d a b c d

      

        (2)

d a d a d a c

a b c d d a b a b c d

     

        (3)

Cộng vế bất đẳng thức ta có :

a b b c c d d a a b c b c d c d a d a b

   

    

        (đpcm)

5.Phƣơng pháp 5:Dùng bất đẳng thức tam giác

Lƣu ý: Nếu a;b;clà số đo ba cạnh tam giác : a; b; c > Và |b-c| < a < b+c ; |a-c| < b < a+c ; |a-b| < c < b+a

(87)

20 CHUYÊN ĐỀ BỒI DƢỠNG TOÁN Cho a; b; clà số đo ba cạnh tam giác chứng minh a, a2 + b2 + c2 < 2(ab + bc + ac)

b, abc > (a+b-c).(b+c-a).(c+a-b) Giải

a)Vì a,b,c số đo cạnh tam giác nên ta có               b a c c a b c b a 0             ) ( ) ( ) ( 2 b a c c c a b b c b a a

Cộng vế bất đẳng thức ta có a2 + b2 + c2 < 2(ab + bc + ac) b) Ta có a > b-c   2

) (b c a

a    > b > a-c  2

) (c a b

b    > c > a-b  c2 c2(ab)2 0

Nhân vế bất đẳng thức ta đƣợc: 2 2  2  2  2

a b c a  b c   b  c a   c  a b 

2   2  2 2      

a b c a b c b c a c a b abc a b c b c a c a b

               

Ví dụ2: (đổi biến số)

Cho a,b,c ba cạnh tam giác Chứng minh

2     

a b

c a c b c b a (1) Đặt x= b + c ; y= c + a ;z = a + b ta có a =

2 x z y 

; b =

2 y x z 

; c =

2 z y x 

ta có (1) 

z z y x y y x z x x z y 2        

   1  1  13

z y z x y z y x x z x y

(  )(  )(  )6

z y y z z x x z y x x y

Bđt đúng? Ví dụ 3: (đổi biến số)

Cho a, b, c > a + b + c <1 Chứng minh : 2 2

2     

ab c

ac b bc

a (1)

Giải: Đặt x = a2 2bc ; y = b22ac ; z = c2 2ab Ta có xyzabc2 1

(1)  111 9

z y

(88)

Theo bất đẳng thức Côsi ta có: 

 y z

x 3.3 xyz   

z y x 1

3 .3

xyz   

1 1           z y x z y x

6) phƣơng pháp làm trội :

Chứng minh BĐT sau :

a) 1 1 1.33.5 (2n1).(2n1)2

b) 1 1.2 1.2.3 1.2.3 n

    

Giải : a) Ta có :

     

2 (2 1)

1 1 1

2 2 (2 1).(2 1) 2

k k

n n k k k k

    

    

       

Cho n chạy từ đến k Sau cộng lại ta có

1 1 1.3 3.5 (2n 1).(2n 1) 2n

 

      

     (đpcm)

b) Ta có :

 

1 1 1

1

1.2 1.2.3 1.2.3 n 1.2 1.2.3 n n

        

< 1 1 1 2

2 n n n

     

          

      (đpcm)

Bài tập nhà:

1) Chứng minh rằng: x2

+ y2

+ z2

+3  (x + y + z) HD: Ta xét hiệu: x2

+ y2

+ z2

+3 – 2( x+ y +z ) = x2

- 2x + + y2

-2y +1 + z2

-2z +1 2) Cho a ,b,c số đo ba cạnh tam giác Chứng minh : a b c

b c c a a b

   

  

(HD: a a a 2a

b c a b c a b c

 

    

a a

b c  a b c  )

3) < 1 1

n + 1 n + 2  2n + 1  3n 3n + <

áp dụng phƣơng pháp làm trội

4) Cho a, b, c > Chứng minh bc ac ab

(89)

20 CHUYÊN ĐỀ BỒI DƢỠNG TOÁN HD: bc ac

a  b = c

b a a b

  

 

   2c;

ac ab b  c ? ;

bc ab a  c ?

CHUYÊN ĐỀ 17 – VẼ ĐƢỜNG THẲNG SONG SONG ĐỂ TẠO THÀNH CÁC CẶP ĐOẠN THẲNG TỶ LỆ

A.Phƣơng pháp:

Trong tập vận dụng định lí Talét Nhiều ta cần vẽ thêm đƣờng phlà đƣờng thẳng song song với đƣờng thẳng cho trƣớc, Đây cách vẽ đƣờng phụ ïhay dùng, nhờ mà tạo thành đƣợc cặp đoạn thẳng tỉ lệ

B Các ví dụ: 1) Ví dụ 1:

Trên cạnh BC, CA, AB tam giác ABC, lấy tƣơng ứng điểm P, Q, R cho ba đƣờng thẳng AP, BQ, CR cắt điểm

Chứng minh: AR BP CQ

RB PC QA (Định lí Cê – va)

Giải

Qua A kẻ đƣờng thẳng song song với BC cắt đƣờng thẳng CR, BQ E, F Gọi O giao điểm AP, BQ, CR

ARE BRC  AR = AE RB BC (a)

BOP FOA  BP = OP FA OA (1)

POC AOE  PC = PO

AE AO (2)

Từ (1) (2) suy ra: BP = PC BP FA FA AE PC AE (b)

AQF CQB  CQ = BC AQ FA (c)

Nhân (a), (b), (c) vế theo vế ta có: AR BP CQ AE FA BC RB PC QA BC AE FA

O F E

R Q

C P

(90)

* Đảo lại: Nếu AR BP CQ

RB PC QA  bai đƣờng thẳng AP, BQ, CR đồng quy 2) Ví dụ 2:

Một đƣờng thăng cắt cạnh( phần kéo dài cạnh) tam giác ABC P, Q, R

Chứng minh rằng: RB.QA.PC

RA.CQ.BP (Định lí Mê-nê-la-uýt)

Giải:

Qua A kẻ đƣờng thẳng song song với BC cắt PR E Ta có

RAE RBP  RB = BP RA AE (a)

AQE CQP  QA = AE QC CP (b)

Nhân vế theo vế đẳng thức (a) (b) ta có

RB QA BP AE

=

RA QC AE CP (1)

Nhân hai vế đẳng thức (1) với PC

BP ta có:

RB PC QA BP AE PC

=

RA BP QC AE CP BP

Đảo lại: Nếu RB.QA.PC

RA.CQ.BP  ba điểm P, Q, R thẳng hàng

3) Ví dụ 3:

Cho tam giác ABC, trung tuyến AM Gọi I điểm cạnh BC Đƣờng thẳng qua I song song với AC cắt AB K; đƣờng thẳng qua I song song với AB cắt AC, AM theo thứ tự D, E Chứng minh DE = BK

Giải

Qua M kẻ MN // IE (N AC).Ta có:

DE AE DE MN

=

MN ANAE  AN (1)

MN // IE, mà MB = MC  AN = CN (2) Từ (1) (2) suy DE MN

AE  CN (3)

N D

I M E

K

C B

(91)

20 CHUYÊN ĐỀ BỒI DƢỠNG TỐN Ta lại có MN CN MN AB

AB  AC CN  AC(4)

Từ (4) (5) suy DE AB

AE AC (a)

Tƣơng tự ta có: BK AB

KI  AC (6)

Vì KI // AC, IE // AC nên tứ giác AKIE hình bình hành nên KI = AE (7)

Từ (6) (7) suy BK BK AB

KI  AE AC (b)

Từ (a) (b) suy DE BK

AE  AE  DE = BK 4) Ví dụ 4:

Đƣờng thẳng qua trung điểm cạnh đối AB, CD tứ giác ABCD cắt đƣờng thẳng AD, BC theo thứ tự I, K Chứng minh: IA KC = ID KB

Giải

Gọi M, N theo thứ tự trung điểm AB, CD Ta có AM = BM; DN = CN

Vẽ AE, BF lần lƣợt song song với CD

AME = BMF (g.c.g)  AE = BF Theo định lí Talét ta có: IA = AE BF

ID DNCN (1)

Củng theo định lí Talét ta có: KB = BF KC CN (2)

Từ (1) (2) suy IA =KB

ID KC  IA KC = ID KB 5) Ví dụ 5:

Cho xOy, điểm A, B theo thứ tự chuyển động tia Ox, Oy cho

1 1

+

OA OB k (k số) Chứng minh AB qua điểm cố định F

E I K

M

N

D C

B

A

E R

Q

C P

(92)

Giải

Vẽ tia phân giác Oz xOy cắt AB C vẽ CD // OA (D  OB)  DOC = DCO = AOC   

COD cân D  DO = DC

Theo định lí Talét ta có CD = BD CD OB - CD OA OBOA  OB

 CD CD 1 1 OAOB  OAOBCD (1)

Theo giả thiết + 1 OA OBk (2)

Từ (1) (2) suy CD = k , không đổi

Vậy AB qua điểm cố định C cho CD = k CD // Ox , D  OB

6) Ví dụ 6:

Cho điểm M di động đáy nhỏ AB hình thang ABCD, Gọi O giao điểm hai cạnh bên DA, CB Gọi G giao điểm OA CM, H

là giao điểm OB DM Chứng minh rằng: Khi M di động AB tổng OG + OH

GD HC không đổi

Giải

Qua O kẻ đƣờng thẳng song với AB cắt CM, DM theo thứ tự I K Theo định lí Talét ta có:

OG OI

GD CD;

OH OK

HC  CD 

OG OH OI OK IK

+

GD HC CDCD CD OG OH IK

+

GD HC CD

  (1)

Qua M vẽ đƣờng thẳng vng góc với AB cắt IK, CD theo thứ tự P Q, ta có:

IK MP FO

CD  MQ MQ không đổi FO khoảng cách từ O đến AB, MQ đƣờng cao hình

thang nên khơng đổi (2)

Từ (1) (2) suy OG + OH FO

GD HC MQ không đổi

Q P

F

K I

H G

M O

D C

B A

z

O

y

x D

C B

(93)

20 CHUYÊN ĐỀ BỒI DƢỠNG TỐN

7) Ví dụ 7:

Cho tam giác ABC (AB < AC), phân giác AD Trên AB lấy điểm M, AC lấy điểm N cho BM = CN, gọi giao điểm CM BN O, Từ O vẽ đƣờng thẳng song song với AD cắt AC, AB E F

Chứng minh rằng: AB = CF; BE = CA Giải

AD phân giác nên BAD = DAF  

EI // AD  BAD = AEF   (góc đồng vị)

Mà DAF OFC (đồng vị); AFE = OFC   (đối đỉnh) Suy AEF AFE  AFE cân A  AE =AF (a) Aùp dụng định lí Talét vào ACD , với I giao điểm EF với BC ta có CF = CI CF CA

CA CD CI  CD (1)

AD phân giác BAC nên CA BA

CD  BD (2)

Từ (1) (2) suy CF BA

CI BD (3)

Kẻ đƣờng cao AG AFE BP // AG (P AD); CQ // AG (Q OI) BPD = CQI  = 900

Gọi trung điểm BC K, ta có BPK = CQK (g.c.g)  CQ = BP  BPD = CQI (g.c.g)  CI = BD (4)

Thay (4) vào (3) ta có CF BA

BD BD  CF = BA (b)

Từ (a) (b) suy BE = CA

Bài tập nhà

1) Cho tam giác ABC Điểm D chia BC theo tỉ số : 2, điểm O chia AD theo tỉ số : gọi K giao điểm BO AC Chứng minh KA

KC không đổi

G

P O K

I N

D Q

C B

M A

(94)

94

2) Cho tam giác ABC (AB > AC) Lấy điểm D, E tuỳ ý thứ tự thuộc cạnh AB, AC cho BD = CE Gọi giao điểm DE, BC K, chứng minh :

Tỉ số KE

KD không đổi D, E thay đổi AB, AC

(HD: Vẽ DG // EC (G  BC)

CHUYÊN ĐỀ 18 – BỔ ĐỀ HÌNH THANG VÀ CHÙM ĐƢỜNG THẲNG ĐỒNG QUY

A Kiến thức

1) Bổ đề hình thang:

“Trong hình thang có hai đáy khơng nhau, đƣờng thẳng qua giao điểm đƣờng chéo qua giao điểm đƣờng thẳng chứa hai cạnh bên qua trung điểm hai đáy”

Chứng minh:

Gọi giao điểm AB, CD H, AC, BD G, trung điểm AD, BC E F Nối EG, FG, ta có: ADG CBG (g.g) , nên :

AD AG 2AE AG AE AG CB  CG  2CF  CG  CF CG (1)

Ta lại có : EAG FCG (SL ) (2)

Từ (1) (2) suy : AEG CFG (c.g.c) Do đó: AGE CGF E , G , H thẳng hàng (3) Tƣơng tự, ta có: AEH BFHAHE BHF

 H , E , F thẳng hàng (4)

Tõừ (3) (4) suy : H , E , G , F thẳng hàng

2) Chùm đƣờng thẳng đồng quy:

// //

/ /

H

G E

F D

C B

A

O

n m

A' B' C'

C B

(95)

20 CHUYÊN ĐỀ BỒI DƢỠNG TOÁN

Nếu đƣờng thẳng đồng quy cắt hai đƣờng thẳng song song chúng định hai đƣờng thẳng song song đoạn thẳng tƣơng ứng tỉ lệ

Nếu m // n, ba đƣờng thẳng a, b, c đồng quy O chúng cắt m A, B, C cắt n A’, B’, C’

AB BC AC

=

A'B' B'C'A'C'

AB A'B' AB A'B' = ;

BC B'C' AC A'C'

* Đảo lại:

+ Nếu ba đƣờng thẳng có hai đƣờng thẳng cắt nhau, định hai đƣờng thẳng song song cặp đoạn thẳng tƣơng ứng tỉ lệ ba đƣờng thẳng đồng quy

+ Nếu hai đƣờng thẳng bị cắt ba đƣờng thẳng đồng quy tạo thành cặp đoạn thẳng tƣơng ứng tỉ lệ chúng song song với

B Aùp dụng: 1) Bài 1:

Cho tứ giác ABCD có M trung điểm CD, N trung điểm CB Biết AM, AN cắt BD thành ba đoạn Chứng minh ABCD hình

bình hành Giải

Gọi E, F giao điểm AM, AN với BD; G, H giao điểm MN với AD, BD

MN // BC (MN đƣờng trung bình BCD)

 Tứ giác HBFM hình thang có hai cạnh bên địng quy A, N trung điểm đáy BF nên theo bổ đề hình thang N trung điểm đáy MH

MN = NH (1)

Tƣơng tự : hình thang CDEN M trung điểm GN  GM = MN (2) Từ (1) (2) suy GM = MN = NH

Ta có BNH = CNM (c.g.c)  BHN = CMN   BH // CM hay AB // CD (a) Tƣơng tự: GDM = NCM (c.g.c)  DGM = CNM   GD // CN hay AD // CB (b)

H

G

F

E

N

M D

C B

(96)

Từ (a) (b) suy tứ giác ABCD có cặp cạnh đối song song nên hình bình hành

2) Bài 2:

Cho ABC có ba góc nhọn, trực tâm H, đƣờng thẳng qua H cắt AB, AC thứ tự tạ P, Q cho HP = HQ Gọi M trung điểm BC Chứng minh: HM PQ

Giải

Gọi giao điểm AH BC I Từ C kẻ CN // PQ (N AB),

ta chứng minh MH CN  HM PQ

Tứ giác CNPQ hình thang, có H trung điểm PQ, hai cạnh bên NP CQ đồng quy A nên K trung điểm CN

 MK đƣờng trung bình BCN  MK // CN  MK // AB (1) H trực tâm ABC nên CHA B (2)

Từ (1) (2) suy MK CH  MK đƣờng cao củaCHK (3) Từ AH BC  MCHK  MI đƣờng cao CHK (4)

Từ (3) (4) suy M trực tâm CHK MHCN  MHPQ

3) 3:

Cho hình chữ nhật ABCD có M, N thứ tự trung điểm AD, BC Gọi E điểm thuộc tia đối tia DC, K giao điểm EM AC

Chứng minh rằng: NM tia phân giác KNE

Giải

Gọi H giao điểm KN DC, giao điểm AC MN I IM = IN Ta có: MN // CD (MN đƣờng trung bình hình chữ nhật ABCD)

 Tứ giác EMNH hình thang có hai cạnh bên EM HN đồng quy K I trung điểm MN nên C trung điểm EH

I K N

M

Q P

H

C B

(97)

20 CHUN ĐỀ BỒI DƢỠNG TỐN Trong ENH NC vừa đƣờng cao, vừa đƣờng

trung tuyến nên ENH cân N  NC tia phân giác ENH mà NC MN (Do NM BC – MN // AB) 

NM tia phân giác góc ngồi N ENH Vậy NM tia phân giác KNE

Bài 4:

Trên cạnh BC = cm hình vng ABCD lấy điểm E cho BE = cm Trên tia đối tia CD lấy điểm F cho CF = cm Gọi M giao

điểm AE BF Tính AMC

Giải

Gọi giao điểm CM AB H, AM DF G Ta có: BH = AB BH

CF FG   FG

Ta lại có AB = BE = CG = 2AB = 12 cm CG EC  2

 FG = cm  BH BH = cm

3  9  BH = BE

BAE = BCH (c.g.c)  BAE = BCH   mà BAE + BEA   = 900

Mặt khác BEA = MEC ; MCE = BCH      MEC + MCE   = 900  AMC = 900 Bài 5:

Cho tứ giác ABCD Qua điểm E thuộc AB, H thuộc AC vẽ đƣờng thẳng song song với BD, cắt cạnh lại tứ giác F, G

a) Có thể kết luận đƣờng thẳng EH, AC, FG

b) Gọi O giao điểm AC BD, cho biết OB = OD Chứng minh ba đƣờng thẳng EG, FH, AC đồng quy

Giải

a) Nếu EH // AC EH // AC // FG

Nếu EH AC không song song EH, AC, FG đồng quy

// //

I

H E

N M

K

D C

B A

H

M

G F

E

D C

(98)

20 CHUYÊN ĐỀ BỒI DƢỠNG TOÁN Trong hình thang DFEB có hai cạnh bên DF, BE đồng quy A OB = OD nên theo bổ đề hình thang M trung điểm EF

Tƣơng tự: N trung điểm GH Ta có ME = MF

GN HN nên ba đƣờng thẳng EG, FH, AC đồng

quy O

O

H

G F

E

N M

D C

(99)

20 CHUYÊN ĐỀ BỒI DƢỠNG TOÁN

CHUYÊN ĐỀ 19 – TÌM GIÁ TRỊ LỚN NHẤT, NHỎ NHẤT CỦA MỘT BIỂU THỨC

A Giá trị lớn nhất, giá trị nhỏ biểu thức

1) Khái niệm: Nếu với giá trị biến thuộc khoảng xác định mà giá trị biểu thức A luôn lớn (nhỏ bằng) số k tồn giá trị biến để A có giá trị k k gọi giá trị nhỏ (giá trị lớn nhất) biểu thức A ứng với giá trị biến thuộc khoảng xác định nói

2) Phƣơng pháp

a) Để tìm giá trị nhỏ A, ta cần: + Chứng minh A  k với k số

+ Chỉ dấ “=” xẩy với giá trị biến b) Để tìm giá trị lớn A, ta cần:

+ Chứng minh A  k với k số

+ Chỉ dấ “=” xẩy với giá trị biến

Kí hiệu : A giá trị nhỏ A; max A giá trị lớn A

B.Các tập tìmGiá trị lớn nhất, giá trị nhỏ biểu thức I) Dạng 1: Tam thức bậc hai

Ví dụ :

a) Tìm giá trị nhỏ A = 2x2 – 8x + b) Tìm giá trị lớn B = -5x2 – 4x + Giải

a) A = 2(x2 – 4x + 4) – = 2(x – 2)2 –  - A = -  x =

b) B = - 5(x2 +

5x) + = - 5(x

+ 2.x.2

5 + 25) +

9 =

9

5 - 5(x + 5)

2 

max B =

5  x =

(100)

b) Ví dụ 2: Cho tam thức bậc hai P(x) = a x2 + bx + c a) Tìm P a >

b) Tìm max P a < Giải

Ta có: P = a(x2 + b

a x) + c = a(x + b 2a )

2

+ (c -

2

b 4a )

Đặt c - b

4a = k Do (x + b 2a )

2 

nên: a) Nếu a > a(x + b

2a ) 

P  k  P = k  x = - b

2a

b) Nếu a < a(x + b

2a ) 

P  k  max P = k  x = - b

2a II Dạng 2: Đa thức có dấu giá trị tuyệt đối

1) Ví dụ 1: Tìm giá trị nhỏ a) A = (3x – 1)2 – 3x - +

đặt 3x - = y A = y2 – 4y + = (y – 2)2 + 

min A =  y =  3x - = 

x = 3x - =

1 3x - = - x = -

3

 

 

 

b) B = x - + x -

B = x - + x - = B = x - + - x  x - + - x =  B =  (x – 2)(3 – x)    x 

2) Ví dụ 2: Tìm GTNN C = 2

x - x +  x - x -

Ta có C = 2

x - x +  x - x - = 2 2

x - x +  + x - x  x - x + + + x - x = C = (x2 – x + 1)(2 + x – x2)   + x – x2   x2 – x –  (x + 1)(x – 2)   -  x 

3) Ví dụ 3:

(101)

20 CHUYÊN ĐỀ BỒI DƢỠNG TOÁN Và x          2 x x x x x = (2)

Vậy T = |x-1| + |x-2| +|x-3| + |x-4|  + = Ta có từ (1)  Dấu xảy 1 x

(2)  Dấu xảy 2 x

Vậy T có giá trị nhỏ 2 x III.Dạng 3: Đa thức bậc cao

1) Ví dụ 1: Tìm giá trị nhỏ

a) A = x(x – 3)(x – 4)(x – 7) = (x2 – 7x)( x2 – 7x + 12) Đặt x2

– 7x + A = (y – 6)(y + 6) = y2 – 36  - 36

Min A = - 36  y =  x2 – 7x + = (x – 1)(x – 6) = x = x = b) B = 2x2 + y2 – 2xy – 2x + = (x2 – 2xy + y2) + (x2 – 2x + 1) +

= (x – y)2 + (x – 1)2 +   x - y = x = y = x - =

 

 

c) C = x2 + xy + y2 – 3x – 3y = x2 – 2x + y2 – 2y + xy – x – y Ta có C + = (x2 – 2x + 1) + (y2 – 2y + 1) + (xy – x – y + 1) = (x – 1)2 + (y – 1)2 + (x – 1)(y – 1) Đặt x – = a; y – = b C + = a2 + b2 + ab = (a2 + 2.a.b

2 +

2

b ) +

2

3b

4 = (a + b 2)

2

+

2

3b

4 

Min (C + 3) = hay C = -  a = b =  x = y = 2) Ví dụ 2: Tìm giá trị nhỏ

a) C = (x + 8)4 + (x + 6)4

Đặt x + = y  C = (y + 1)4 + (y – 1)4 = y4 + 4y3 + 6y2 + 4y + + y4 - 4y3 + 6y2 - 4y + = 2y4 + 12y2 +   A =  y =  x = -

b) D = x4 – 6x3 + 10x2 – 6x + = (x4 – 6x3 + 9x2 ) + (x2 – 6x + 9) = (x2 – 3x)2 + (x – 3)2   D =  x =

IV Dạng phân thức:

1 Phân thức có tử số, mẫu tam thức bậc hai

(102)

Ví dụ : Tìm GTNN A = 2

6x - - 9x = 2

- 2

9x - 6x + (3x - 1)

 

Vì (3x – 1)2   (3x – 1)2 +   2 22

(3x - 1) 4 (3x - 1) 4

 

  

   A  -

1

min A = -1

2  3x – =  x =

2 Phân thức có mẫu bình phƣơng nhị thức a) Ví dụ 1: Tìm GTNN A =

2

3x - 8x + x - 2x +

+) Cách 1: Tách tử thành nhóm có nhân tử chung với mẫu A =

2

2 2

3x - 8x + 3(x - 2x + 1) - 2(x - 1) +

=

x - 2x + (x - 1)  x - 1(x - 1) Đặt y =

x - Thì

A = – 2y + y2 = (y – 1)2 +   A =  y = 

x - =  x =

+) Cách 2: Viết biểu thức A thành tổng số với phân thức không âm A =

2 2

2 2

3x - 8x + 2(x - 2x + 1) + (x - 4x + 4) (x - 2)

= 2

x - 2x + (x - 1)  (x - 1) 

 A =  x – =  x = b) Ví dụ 2: Tìm GTLN B = 2 x

x 20x + 100

Ta có B = 2 x x 2

x 20x + 100(x + 10) Đặt y =

x + 10  x =

10 y

B = (1 10 y ).y

2

= - 10y2 + y = - 10(y2 – 2.y

20y + 400) +

1

40 = - 10

2 y - 10       + 40 

1 40

Max B =

40 

1 y -

10 =  y =

10  x = 10

c) Ví dụ 3: Tìm GTNN C =

2

2

x + y x + 2xy + y

Ta có: C =

2

2 2

2 2

1

(x + y) (x - y)

x + y 2 1 (x - y)

x + 2xy + y (x + y) 2 (x + y)

  

 

     A =

(103)

20 CHUN ĐỀ BỒI DƢỠNG TỐN a)Ví dụ : Tìm GTNN, GTLN (Cực trị) A = - 4x2

x 1

Ta có: A =

2 2

2 2

3 - 4x (4x 4x 4) (x 1) (x - 2)

1

x x x

   

    

    A = -  x =

Ta lại có: A =

2 2

2 2

3 - 4x (4x 4) (4x + 4x + 1) (2x 1)

4

x x x

  

   

    max A =  x =

1

C Tìm GTNN, GTLN biểu thức biết quan hệ biến 1) Ví dụ 1: Cho x + y = Tìm GTNN A = x3 + y3 + xy

Ta có A = (x + y)(x2 – xy + y2) + xy = x2 + y2 (vì x + y = 1)

a) Cách 1: Biểu thị ẩn qua ẩn kia, đƣa tam thức bậc hai Từ x + y =  x = – y

nên A = (1 – y)2 + y2 = 2(y2 – y) + = 2(y2 – 2.y.1

2 + 4) +

1 =

2

1 1

y - +

2 2

  

 

 

Vậy A =

2  x = y =

b) Cách 2: Sử dụng đk cho, làm xuất biểu thức có chứa A

Từ x + y =  x2 + 2xy + y2 = 1(1) Mặt khác (x – y)2   x2 – 2xy + y2  (2) Cộng (1) với (2) vế theo vế, ta có:

2(x2 + y2)   x2 + y2 

2  A =

2  x = y = 2)Ví dụ 2: Cho x + y + z =

a) Tìm GTNN A = x2 + y2 + z2 b) Tìm GTLN B = xy + yz + xz

Từ Cho x + y + z =  Cho (x + y + z)2 =  x2 + y2 + z2 + 2(xy + yz + xz) = (1) Ta có x2

+ y2

+ z2

- xy – yz – zx =

2

.2 ( x2

+ y2

+ z2

- xy – yz – zx) =

2

1 2

(x y) (x z) (y z)

      

    x2 + y2 + z2  xy+ yz + zx (2)

Đẳng thức xẩy x = y = z a) Từ (1) (2) suy

(104)

 x2 + y2 + z2   A =  x = y = z = b) Từ (1) (2) suy

= x2 + y2 + z2 + 2(xy + yz + xz)  xy+ yz + zx + 2(xy + yz + xz) = 3(xy+ yz + zx)  xy+ yz + zx   max B =  x = y = z =

3) Ví dụ 3:

Tìm giá trị lớn S = xyz.(x+y).(y+z).(z+x) với x,y,z > x + y + z = Vì x,y,z > ,áp dụng BĐT Cơsi ta có: x+ y + z 33 xyz 1

3 27

xyz xyz

   

áp dụng bất đẳng thức Côsi cho x+y ; y+z ; x+z ta có

xy . yz . zx33xy . yz . xz  2 33xy . yz . zx

Dấu xảy x = y = z =1

3  S 

8

27 27 729

Vậy S có giá trị lớn

729 x = y = z =

4) Ví dụ 4: Cho xy + yz + zx = Tìm giá trị nhỏ 4

xyz

Áp dụng BĐT Bunhiacốpski cho số (x,y,z) ;(x,y,z) Ta có  2  2 22

xyyzzxxyz  1 x2y2z22 (1) áp dụng BĐT Bunhiacốpski cho ( 2

, ,

x y z ) (1,1,1)

Ta có 2 2 2 4 2 2 4

(xyz ) (1  1 )(xyz )(xyz ) 3(xyz )

Từ (1) (2) 4

1 3(x y z )

    4

3

x y z

   

Vậy 4

xyz có giá trị nhỏ

3 x= y = z = 3

D Một số ý:

1) Khi tìm GTNN, GTLN ta đổi biến

Ví dụ : Khi tìm GTNN A =(x – 1)2 + (x – 3)2 , ta đặt x – = y A = (y + 1)2 + (y – 1)2 = 2y2 +  2…

(105)

20 CHUYÊN ĐỀ BỒI DƢỠNG TOÁN +) -A lớn  A nhỏ ; +)

Blớn  B nhỏ (với B > 0)

+) C lớn  C2 lớn Ví dụ: Tìm cực trị A =

 

4 2

x + x +

a) Ta có A > nên A nhỏ

A lớn nhất, ta có  2 2

2

4

x +

1 2x

1

A  x +  x + 1 

A =  x =  max A =  x =

b) Ta có (x2 – 1)2   x4 - 2x2 +   x4 +  2x2 (Dấu xẩy x2 = 1) Vì x4 + > 

2

2x

x +  

2

2x

1 1

x +

     max

A =  x

=  A =

2  x = 1

3) Nhiều ta tìm cực trị biểu thức khoảng biến, sau so sámh cực trị để để tìm GTNN, GTLN tồn tập xác định biến

Ví dụ: Tìm GTLN B = y

5 - (x + y)

a) xét x + y 

- Nếu x = A = - Nếu  y  A  - Nếu y = x = A =

b) xét x + y  A 

So sánh giá trị A, ta thấy max A =  x = 0; y = 4) Sử dụng bất đẳng thức

Ví dụ: Tìm GTLN A = 2x + 3y biết x2 + y2 = 52

Aùp dụng Bđt Bunhiacốpxki: (a x + by)2  (a2 + b2)(x2 + y2) cho số 2, x , 3, y ta có: (2x + 3y)2  (22 + 32)(x2 + y2) = (4 + 9).52 = 262  2x + 3y  26

Max A = 26 x = y

2

 y = 3x

2  x

+ y2 = x2 +

2

3x

 

 

  = 52  13x

2

(106)

Vậy: Ma x A = 26  x = 4; y = x = - 4; y = -

5) Hai số có tổng khơng đổi tích chúng lớn chúng Hai số có tích khơng đổi tổng chúng lớn chúng

a)Ví dụ 1: Tìm GTLN A = (x2 – 3x + 1)(21 + 3x – x2)

Vì (x2 – 3x + 1) + (21 + 3x – x2) = 22 khơng đổi nên tích (x2 – 3x + 1)(21 + 3x – x2) lớn x2 – 3x + = 21 + 3x – x2  x2 – 3x – 10 =  x = x = - Khi A = 11 11 = 121  Max A = 121  x = x = -

b) Ví dụ 2: Tìm GTNN B = (x + 4)(x + 9)

x

Ta có: B =

2

(x + 4)(x + 9) x 13x + 36 36 x + 13

x x x

  

Vì số x 36

x có tích x 36

x = 36 không đổi nên

36 x +

x nhỏ x = 36

x  x =

 A = x + 36 13

x  nhỏ A = 25  x =

6)Trong tìm cực trị cần tồn giá trị biến để xẩy đẳng thức không cần giá trị để xẩy đẳng thức

Ví dụ: Tìm GTNN A = m n

11 5

Ta thấy 11m tận 1, 5n tận

Nếu 11m > 5n A tận 6, 11m < 5n A tận

(107)

20 CHUYÊN ĐỀ BỒI DƢỠNG TOÁN

CHUYÊN ĐỀ 20 – PHƢƠNG TRÌNH NGHIỆM NGUYÊN  - PHƯƠNG PHÁP 1: Phƣơng pháp đƣa dạng tổng

 Phƣơng pháp: Phương pháp thường sử dụng với phương trình có biểu thức chứa ẩn viết dạng tổng bình phương

- Biến đổi phƣơng trình dạng vế tổng bình phƣơng biểu thức chứa ẩn; vế cịn lại tổng bình phƣơng số nguyên (số số hạng hai vế nhau)

Các ví dụ minh hoạ:

- Ví dụ 1: Tìm x;yZthoả mãn: 5x24xyy2 169 (1) (1)4x24xyy2x2 144251690   

 

2 2

2 2

2 144 25

2 169

x y x

x y x

    

 

   



Từ (I) ta có: Tƣơng tự từ (II) ta có:

    2 2 2 2 5 12 ; 22 12 12 ; 19 29 12 x x x y y y x x x x y y y x                                                2 2 2 13 13 13 26 13 x x y y x x x y y x                            

Vậy              

           

5; ; 5; 22 ; 5; ; 5; 22 ; 12; 19 ; 12; 29 ,

12;19 ; 12; 29 ; 0;13 ; 0; 13 ; 13; 26 ; 13; 26

x y        

    

 

 

Ví dụ 2: Tìm x;yZthoả mãn: 2

8 xy   x y (2)

(2) 2 2   2 2 2

4x 4x 4y 4y 32 4x 4x 4y 4y 34 2x 2y

                          2 2 2 2

2 2;

3;

2

2 3;

2;

2

x x x

y y

y

x x x

y y y                                   

Vậy     x y;  2;3 ; 2; ;  1;3 ;  1; ; 3;2 ; 3; ;      2;2 ;  2; 

Ví dụ 3: Tìm x;yZthoả mãn: 3

91 xy  (1)

(108)

(1)   2

91.1 13.7 x y x xy y

      (Vì  2

0 xxyy  )

     

 

2

2

2

1 6 5

;

91

91.1

91

1

x y x x

x xy y y y

x y x xy y

x y

VN x xy y

       

        

  

     

  

 

   

  

Ví dụ 4: Tìm x;yZthoả mãn: 2

0 x  x y  (2)

   2   

2 2

0 4 2 2 1

2 1

2 1

2 1

2 1

x x y x x y x y x y x xy

x y x

x y y

x y x

x y y

                

     

 

    

 

        

      

 

Vậy:     x y;  0;0 ; 1;0

 - PHƯƠNG PHÁP 2: Phƣơng pháp cực hạn

 Phƣơng pháp: Phương pháp thường sử dụng với phương trình đối xứng

- Vì phƣơng trình đối xứng nên x y z; ; có vai trị bình đẳng nhƣ Do đó; ta giả thiết x y z; tìm điều kiện nghiệm; loại trừ dần ẩn để có phƣơng trình đơn giản Giải phƣơng trình; dùng phép hốn vị để suy nghiệm

 Ta thƣờng giả thiết 1   x y z

Các ví dụ minh hoạ:

Ví dụ 1: Tìm x y z; ; Z thoả mãn: x  y z x y z (1)

Nhận xét – Tìm hướng giải:

Ta thấy phƣơng trình đối xứng Giả sử 1  x y z Khi đó:

(1)x y z    x y z 3zx y 3 (Vì x y z; ; Z)x y 1; 2;3

* Nếu: x y       1 x y z z(vơ lí) * Nếu: x y   2 x 1;y2;z3

(109)

20 CHUYÊN ĐỀ BỒI DƢỠNG TỐN Vậy: x y z; ; hốn vị 1; 2;3

Ví dụ 2: Tìm x y z; ; Z thoả mãn: 1

x  y z (2)

Nhận xét – Tìm hướng giải: Đây phƣơng trình đối xứng Giả sử 1  x y z Khi đó:

(2) 1 3

2

x x

x y z x

        

Với: x 1 1 y y  1;

y z y

        

.Nếu: y 1

z

   (vơ lí)

.Nếu: y  2 z

Vậy: x y z; ; hoán vị 1; 2; 2

 - PHƯƠNG PHÁP 3: Phƣơng pháp sử dụng tính chất chia hết

Các ví dụ minh hoạ: Ví dụ 1: Tìm x y; Z để:

2

1

x x A

x x  

  nhận giá trị nguyên Ta có:

2

2 2

1 1

1

1 1

x x x x

A

x x x x x x

   

   

      Khi đó:

Để A nhận giá trị nguyên 2

1

x  x nhận giá trị nguyên      1  

1 x x x x U 1;1

         

Vì :  

1 0; 1

1 x

x x x x x

x

 

          

  

Vậy để A nhận giá trị nguyên thì: x0 x 1

Ví dụ 2: Tìm x y; Z thoả mãn: 2

2y x   x y x 2yx y

(2)        

2y x x x y x 1 *

       

(110)

 

2

2 **

1 y x y

x

   

Phƣơng trình có nghiệm ngun  1 (1)  1; 1

x

x U

x x

 

        

  

Ví dụ 3: Tìm x y; Z thoả mãn: 3x 1 y12 (3) Ta có:

(3)3x y12 1 y y 2.3x số lẻ y y; 2là hai số lẻ liên tiếp y y; 2 y y;

     luỹ thừa 3, nên:

 

  

3 *

3

2 **

m

m n

n y

m n x m n

y  

       

   

 Với: m    0; n y 1;x1

 Với: m  1; n 1Từ    

    

3

* ; ** ;

2 y

y y y



    



 ( vơ lí)

Phƣơng trình có nghiệm nguyên:

1 x y

    

 - PHƯƠNG PHÁP 4: Phƣơng pháp sử dụng bất đẳng thức

 Phƣơng pháp: Phương pháp thường sử dụng với phương trình mà hai vế những đa thức có tính biến thiên khác

- Áp dụng bất đẳng thức thƣờng gặp: *Bất đẳng thức Cô – si:

Cho n số không âm: a a a1; 2; 3; ;an Khi đó:

1

1

n n

n

a a a a

a a a a n

    

Dấu “=” xảy  a1 a2 a3 an * Bất đẳng thức Bunhiacôpxki:

Cho 2n số thực: a a a1; 2; 3; ;anb b b1; ; ; ;2 bn Khi đó:

  2  

1.1 2 3 n n n n

a ba ba b  a baa   a a bb   b b Dấu “=” xảy  ai kb ii 1;n

(111)

20 CHUYÊN ĐỀ BỒI DƢỠNG TOÁN

a b a b a b

a b a b       

   

Các ví dụ minh hoạ:

Ví dụ 1: Tìm x y; Z thoả: x y y z z x

zxy  (1)

Áp dụng BĐT Cơ – si Ta có: 3 x y y z z x 3.3 x y y z z x . . 3.3 x y z .

z x y z x y

    

3 x y z . 1 x y z . 1 x y z 1

       

Vậy nghiệm phƣơng trình là: x  y z

Ví dụ 2: Tìm nghiệm ngun phƣơng trình:  2  2 

1

x yxy  (2)

(Tốn Tuổi thơ 2) Theo Bunhiacơpxki,ta có:

 2  2 2 2   2 

1 1 1

x y    xy   xy

Dấu “=” xảy 1

1 1 x y

x y

     

Vậy nghiệm phƣơng trình là: x y

Ví dụ 3: Tìm tất số ngunx thoả mãn:

x  3 x 10 x 101 x 990 x 1000 2004 (3)

Nhận xét – Tìm hướng giải:

Ta nhận thấy: 2104 = + 10 + 101 + 990 + 1000 =101 + 2003 a  a Ta có:(3)  3 x 10  x x 101 x 990  x 1000 2004

3

10 10

101 101 2004 101 2003 101 990 990

1000 1000

x x

x x

a a x x x x

x x

x x

    

  

 

           

   

   

(112)

Với x 10120042003(vơ lí) Vậy nghiệm phƣơng trình là: x  102; 100 

1) Tìm số nguyên x,y,z thoả mãn: 2

3 xyzxyyz

Vì x,y,z số nguyên nên 2

3 xyzxyyz

 

2

2 2

3 3

4

y y

x y z xy y zx xy   yz z

                  

   

 

2

2

3 1

2

y y

x z

   

         

    (*) Mà  

2

2

3 1

2

y y

x z

         

   

    x y, R

 

2

2

3 1

2 y y x z                   1

1

2 1 y x x y y z z                     

Các số x,y,z phải tìm

1 x y z        

PHƯƠNG PHÁP 5: Phƣơng pháp lựa chọn

Phƣơng pháp: Phương pháp sử dụng với phương trình mà ta nhẩm (phát dể dàng) vài giá trị nghiệm

- Trên sở giá trị nghiệm biết Áp dụng tính chất nhƣ chia hết; số dƣ; số phƣơng; chữ số tận … ta chứng tỏ với giá trị khác phƣơng trình vơ nghiệm

Các ví dụ minh hoạ:

Ví dụ 1: Tìm x y; Z thoả mãn:

3

xx   y

Nhận xét – Tìm hướng giải:

Ta thấy với x0;y 1 phƣơng trình đƣợc nghiệm Ta cần chứng minh phƣơng trình vơ nghiệm với x0

+ Với x0;y 1 phƣơng trình đƣợc nghiệm

+ Với x0 Khi đó:

6  2  2

2 4

xx  xx  xx   x   yx  (*) Vì    

1 ;

(113)

20 CHUYÊN ĐỀ BỒI DƢỠNG TOÁN Vậy x0;y 1 nghiệm phƣơng trình

Ví dụ 2: Tìm x y; Z thoả: 2

1 y

x   x  (2)

(Tạp chí Tốn học tuổi trẻ ) Gọi b chữ số tận x ( Với b0;1; 2; ;9 Khi đó:  

1

x  x có chữ số tận là: 1, (*)

Mặt khác:

3 y luỹ thừa bậc lẻ nên có tận (**) Từ (*) (**) suy phƣơng trình vơ nghiệm

Ví dụ 3: Tìm x y; Z thoả mãn: 2

6 13 100 xxyy  (3)

(3)     

  

2 2

2

5 25

25

y

x y

y n n

        

  

 

Do đó: y    5; 4; 3;0;3; 4;5 x 3;9;11;13

Phƣơng trình có nghiệm ngun:   x y;   5;3 ; 4;9 ; 3;11 ; 0;13 ; 3;11 ; 4;9 ; 5;3        

PHƯƠNG PHÁP 6: Phƣơng pháp lùi vô hạn (xuống thang)

Phƣơng pháp: Phương pháp thường sử dụng với phương trình có (n – 1) ẩn mà

hệ số có ước chung khác 1

- Dựa vào tính chất chia hết ta biểu diễn ẩn theo ẩn phụ nhằm “hạ” (giảm bớt) số tự do, để có đƣợc phƣơng trình đơn giản

- Sử dụng linh hoạt phƣơng pháp để giải phƣơng trình

Các ví dụ minh hoạ:

Ví dụ 1: Giải phƣơng trình: 3

3

xyz  (1)

Nhận xét – Tìm hướng giải:

Ta thấy 3  3 3

3 9

xyz   xyz  mà  3

3y 9z

   nên

3

x  Ta có: (1)  3 3

1

3 3 3

x y z x x x x

         

Khi đó: (1)  3 3  3 3

1 1

27x 3y 9z 9x y 3z y y y 3y

             

 3 3

1 1

9x 27y 3z z z y 3z

(114)

* Tiếp tục biểu diễn gọi x y z0; 0; nghiệm (1) 3Ux y z0; 0; 0và

0 0

0x y z; ; 9 Thực thử chọn ta đƣợc: x0  y0 z0 0

(115)

20 CHUYÊN ĐỀ BỒI DƢỠNG TOÁN CÁC BÀI TẬP KHÁC

1/Dùng định nghĩa

1) Cho abc = a3 36 Chứng minh 

3

2

a

b2+c2> ab+bc+ac Giải

Ta có hiệu: 

3

2

a

b2+c2- ab- bc – ac = 

4

2

a

12

2

a

b2+c2- ab- bc – ac

= ( 

4

2

a

b2+c2- ab– ac+ 2bc) + 

12

2

a

3bc =(

2 a

-b- c)2 +

a abc a

12 36

3 

=(

2 a

-b- c)2 +

a abc a

12 36

3 

>0 (vì abc=1 a3 > 36 nên a >0 )

Vậy : 

3

2

a

b2+c2> ab+bc+ac Điều phải chứng minh 2) Chứng minh

a) x4y4z212x.(xy2xz1)

b) với số thực a , b, c ta có : a25b24ab2a6b30

c) a22b22ab2a4b20

Giải :

a) Xét hiệu :

H = x4y4 z212x2y22x22xz2x =  2 22   2 2

1

   

y x z x x

H0 ta có điều phải chứng minh b) Vế trái viết

H = a2b1 2  b12 1

 H > ta có điều phải chứng minh c) vế trái viết

H =   2 2

1  

b b

a

(116)

Ii / Dùng biến đổi tƣơng đƣơng

1) Cho x > y xy =1 Chứng minh :  2

2 2    y x y x Giải :

Ta có x2y2xy22xyxy22 (vì xy = 1)

 x2 y22 xy44.xy24

Do BĐT cần chứng minh tƣơng đƣơng với

 4  2  2

4 x y x y

y

x       xy44xy240   

2 2 x y       

BĐT cuối nên ta có điều phải chứng minh 2) Cho xy  Chứng minh :

xy y

x    

 1 1 2 Giải : Ta có xy y

x    

 1 1

2 

1 1 1 1 2

2 

                

x y y xy

       1 2 2         xy y y xy xy x x xy

      ) ( ) (

2   

     xy y y x y xy x x y x

         1 2       xy y x xy x y

BĐT cuối xy > Vậy ta có điều phải chứng minh Iii / dùng bất đẳng thức phụ

1) Cho a , b, c số thực a + b +c =1 Chứng minh

3

2

2  

c b a

Giải :

áp dụng BĐT BunhiaCôpski cho số (1,1,1) (a,b,c) Ta có   2  2 2

1

1abc    abc   2  2 2

3 a b c c

b

(117)

20 CHUYÊN ĐỀ BỒI DƢỠNG TOÁN  2

2  

c b

a (vì a+b+c =1 ) (đpcm)

2) Cho a,b,c số dƣơng

Chứng minh   1 19

         c b a c b

a (1)

Giải :

(1)  1   1   19

a c a c c b a b c a b a

3 

                       b c c b a c c a a b b a

áp dụng BĐT phụ  2

x y y x

Với x,y > Ta có BĐT cuối ln

Vậy   1 19

         c b a c b

a (đpcm)

Iv / dùng phƣơng pháp bắc cầu

1) Cho < a, b,c <1 Chứng minh : 2a32b32c33a2bb2cc2a

Giải :

Do a <1 

a <1 b <1 Nên 1a2.1b201a2ba2b0

Hay 1a2ba2b (1) Mặt khác <a,b <1 

a

a  ; bb3  1a2a3b3

Vậy a3b31a2b

Tƣơng tự ta có :

a c c a c b c b 3 3 1      

 2a32b32c33a2bb2cc2a (đpcm) 2) So sánh 3111

1714

Giải :

Ta thấy 11

31 < 11  5 11 55 56

32  2 2

Mặt khác 56 4.14  4 14 14 14

2 2  16 17

Vởy 3111

< 1714

(118)

V/ dùng tính chất tỉ số

ví dụ 4: Cho số a,b,c,d bất kỳ, chứng minh rằng: 2 2 2

) ( )

(acbdabcd

Giải: Dùng bất đẳng thức Bunhiacopski ta có ac + bd  2 2

c d b

a  

mà   2 2 2   2

2 ac bd c d b

a d b c

a         a2 b22 a2 b2 c2 d2 c2 d2

 2 2 2

) ( )

Ngày đăng: 01/03/2021, 08:27

Tài liệu cùng người dùng

Tài liệu liên quan